Scenarios Ch.17 Flashcards Preview

GI2- AM > Scenarios Ch.17 > Flashcards

Flashcards in Scenarios Ch.17 Deck (100)
Loading flashcards...
1
Q
  1. A 27-year-old man who was badly burned in an industrial accident requires multiple skin grafting. While in the intensive care unit, he is found to have blood in his stools, and endoscopy confirms the presence of many small ulcers in his stomach. This complication is referred to as (A) aplastic anemia. (B) Curling ulcer. (C) Cushing ulcer. (D) Reye syndrome. (E) Stevens-Johnson syndrome.
A

The answer is B. Punctate ulcers associated with extensive burn injuries are known as Curling ulcers. A similar phenomenon occurs in patients with head trauma, in which the lesions are known as Cushing ulcers. Aplastic anemia can result from an idiosyncratic reaction in patients taking the antibiotic chloramphenicol. Likewise, sulfonamides can cause a necrotizing eruption around mucous membranes in some individuals. Reye syndrome is associated with extensive microvesicular fatty change of the liver in children taking aspirin during an acute viral illness. The Stevens-Johnson syndrome is characterized by erosions and crusts of the lips and oral mucosa as a component of an extensive form of erythema multiforme, a maculopapular, vesiculobullous eruption often related to drugs (such as sulfonamides), neoplasia, or connective tissue disorders.

2
Q
  1. A 73-year-old man develops severe, intractable diarrhea during hospitalization for bacterial pneumonia caused by a multidrug- resistant organism. What organism is most likely responsible for his gastrointestinal symptoms? (A) MRSA (B) Streptococcus pneumoniae (C) Candida (D) Clostridium difficile (E) Clostridium botulinum
A

The answer is D. C. difficile causes diarrhea in patients in whom competing colonic flora has been obliterated by antibiotics. Candidal infections are also common in patients on antibiotics but typically manifest with oral or vaginal candidiasis, rather than diarrhea. MRSA is a growing problem both in the hospital and in the community but does not typically cause diarrhea (although it could well be the cause of the patient’s drug-resistant pneumonia!). C. botulinum causes botulism, a rare but sometimes fatal paralytic disorder, and is not associated with diarrhea.

3
Q
  1. A 60-year-old man presents with hematemesis, melena, guaiac-positive stools, and signs of circulatory collapse. He has a 20-year history of burning midepigastric pain and tenderness relieved by food, milk, or antacids. Also, he has been taking high doses of NSAIDs to relieve the pain of long-standing arthritis. Esophagogastroduodenoscopy reveals a peptic ulcer in the upper duodenum. Which of the following is an important association of duodenal peptic ulcer disease? (A) Barrett esophagus and columnar intestinal metaplasia of esophageal squamous epithelium (B) Evolution into carcinoma as a likely sequela (C) H. pylori infection (D) Hiatal hernia and incompetent lower esophageal sphincter (E) Pernicious anemia and achlorhydria
A
  1. T he answer is C. Of course, the immediate problem in this patient is life-threatening upper gastrointestinal hemorrhage, an important complication of peptic ulcer disease. Peptic ulcer disease occurs most frequently in the first portion of the duodenum, the lesser curvature of the stomach, or the distal esophagus. Duodenal peptic ulcers are associated with hypersecretion of gastric acid and pepsin and are closely related to gastric H. pylori infection. Apparently, H. pylori increases gastric acid secretion and impairs mucosal defenses. Other predisposing factors include aspirin or NSAID intake, smoking, Zollinger- Ellison syndrome, primary hyperparathyroidism, and multiple endocrine neoplasia type I.
4
Q
  1. A 60-year-old Caucasian man with a 5-year history of gastroesophageal reflux disease (GERD) presents with persistent pyrosis (heartburn) and acid regurgitation. He has had similar symptoms for the past 5 years. Because this patient has a long history of GERD, an esophagogastroduodenoscopy is performed to screen for Barrett esophagus, a well-known complication of long-standing GERD. Results reveal that Barrett esophagus is indeed present. Which of the following is true of Barrett esophagus? (A) A biopsy will show a histologic finding of columnar-to-squamous metaplasia. (B) It is a known precursor of adenocarcinoma of the esophagus. (C) It is a known precursor of carcinoma of the stomach. (D) It is a known precursor of squamous cell carcinoma of the esophagus. (E) The most common location is the proximal (upper) third of the esophagus.
A
  1. T he answer is B. Barrett esophagus is columnar metaplasia of the esophageal squamous epithelium (squamous-to-columnar). The columnar epithelium is often of the intestinal type with goblet cells. Barrett esophagus is a complication of long-standing GERD and is a precursor of esophageal adenocarcinoma. The most common location is in the distal (lower) third of the esophagus.
5
Q
  1. A 65-year-old man presents with dysphagia, weight loss, and anorexia. Physical examination is normal. Esophagogastroduodenoscopy with biopsy of an esophageal lesion is performed, revealing squamous cell carcinoma. Which of the following is true regarding this cancer? (A) Cigarette smoking and chronic alcohol use are associated risk factors. (B) Gastroesophageal reflux disease and Barrett esophagus are associated risk factors. (C) Histologic findings include disordered, back-to-back submucosal glands. (D) It most frequently arises in the lower third of the esophagus. (E) This cancer is characterized by an indolent course, and long survival is common.
A
  1. T he answer is A. Squamous cell carcinoma of the esophagus is an aggressive cancer with rapid progression and short survival in all stages of disease. It is most common in subjects with a long-term history of cigarette smoking and alcohol use. The tumor arises most commonly in the upper and middle thirds of the esophagus.
6
Q
  1. A 10-day-old infant presents with projectile vomiting. His mother states that the infant will actively drink his milk, but he forcefully vomits after each feeding. The infant shows signs of failure to thrive, with weight loss, dehydration, and lethargy. Physical examination reveals a firm, nontender, mobile, “olive-shaped” epigastric mass. Which of the following is the most likely diagnosis? (A) Candida esophagitis (B) Congenital pyloric stenosis (C) Esophageal cancer (D) GERD (E) Tracheoesophageal fistula
A
  1. T he answer is B. Congenital pyloric stenosis is caused by hypertrophy of the circular muscular layer of the pylorus, resulting in a palpable mass in the epigastrium. Hypertrophy of the pyloric musculature leads to obstruction and the characteristic projectile vomiting. This condition most commonly occurs in male infants within the first several days to weeks of life.
7
Q
  1. A 25-year-old man presents with lowgrade fever, weight loss, fatigue, crampy abdominal pain, episodic diarrhea, and postprandial bloating. Right lower quadrant tenderness is elicited on palpation of the abdomen. A capsule endoscopy reveals thickening of the terminal ileum, edema, marked luminal narrowing, and a cobblestone appearance of the mucosa. Which of the following is a characteristic of this condition? (A) Additional typical findings include crypt abscesses and pseudopolyps. (B) Inflammation and ulceration limited to mucosa and submucosa with sparing of deeper layers. (C) It can affect any portion of the gastrointestinal tract, but proximal jejunum is most common site of involvement. (D) It can cause fistula formation between loops of affected bowel. (E) It is a benign, self-limited disorder with no complicating sequelae.
A
  1. The answer is D. Crohn disease and ulcerative colitis are the two classic inflammatory bowel diseases. Crohn disease is a chronic inflammatory condition that can affect any part of the gastrointestinal tract from mouth to anus, but most commonly involves the distal ileocecum, small intestine, or colon. Morphologically, Crohn disease manifests as transmural inflammation (involving all layers of the intestinal wall), thickening of involved intestine, linear ulceration, a cobblestone appearance, skip lesions (normal intestine between affected regions), and granulomas. Strictures and fistulae may develop, leading to intestinal obstruction. Crohn disease may lead to carcinoma of the small intestine or colon, but much less commonly than ulcerative colitis.
8
Q
  1. A 70-year-old man presents with fatigue, weight loss, abdominal pain, and overt blood in the stools. A complete blood count reveals anemia with hemoglobin of 10.0 g/dL. A colonoscopy and colon biopsy reveal adenocarcinoma. Which of the following is the most likely predisposing lesion that led to this condition? (A) FAP syndrome (B) Hyperplastic polyp (C) Long-standing ulcerative colitis (D) Peutz-Jeghers polyp (E) Tubular adenoma
A
  1. T he answer is E. Adenocarcinoma of the colon most commonly develops through a progression of mutations in oncogenes and tumor suppressor genes in a multistep process. Normal mucosa evolves into a tubular adenoma with malignant potential, which then further evolves into carcinoma (the adenoma-carcinoma sequence). Carcinoma of the rectosigmoid (left-sided) tends to present as early obstruction, with change in bowel habits and decreased caliber of stool, whereas carcinoma of the right colon (right-sided) tends to present late, with iron deficiency anemia due to chronic blood loss from the lesion.
9
Q
  1. For the past week, a 65-year-old woman has been treated for a severe infection with broad-spectrum antibiotics, and she had recovered well. Over the past day, however, she has developed foul-smelling, voluminous, greenish, watery diarrhea, as well as abdominal pain and fever. She is diagnosed with pseudomembranous colitis. Which of the following is the mechanism associated with this condition? (A) Aggregation of bacterial colonies on the lumen, forming pseudomembranes (B) Bacterial release of exotoxin, inducing necrosis of the mucosa (C) Physical invasion of bacteria into the superficial mucosa, leading to pseudomembrane formation (D) Selective killing of C. difficile bacteria by antibiotics (E) Spread of the previous infection to the colon
A
  1. T he answer is B. Pseudomembranous colitis is caused by overgrowth of C. difficile. This organism produces exotoxin that induces necrosis of the superficial mucosa, leading to pseudomembrane formation. The bacteria itself does not invade the mucosa. This condition most often occurs in patients with a history of broad-spectrum antibiotic use, because elimination of normal intestinal flora promotes overgrowth of C. difficile.
10
Q
  1. An elderly woman with chronic constipation dies of a stroke and comes to autopsy. The figure illustrates a portion of her colon. The lesions shown in the figure (A) can be complicated by inflammation, perforation, and peritonitis. (B) are most likely related to a high-fiber diet. (C) most frequently occur high on the right side of the colon. (D) occur most often in teenagers.
A
  1. T he answer is A. The illustration demonstrates diverticulosis of the colon (openings shown by arrows). These lesions are most common in older persons and are found most often in the sigmoid. The incidence of disease is increased in populations that consume low-fiber diets. Although most often asymptomatic, diverticula may become the site of acute inflammation (diverticulitis), sometimes with life-threatening complications, such as perforation and peritonitis.
11
Q
  1. A 69-year-old man was seen for vague abdominal distress. The gastric lesion shown in the figure was resected following initial endoscopic discovery. Which of the following statements about this condition is correct? (A) It has been decreasing in frequency over the past several decades. (B) It is more frequent in Japan than in the United States. (C) It is related to the use of nitrites as food preservatives. (D) It may result in Krukenberg tumors. (E) It will most likely heal with conservative management.
A
  1. T he answer is E. The illustration shows a chronic gastric peptic ulcer with characteristic radiating folds of the gastric mucosa starting at the ulcer margins. The lesion has a smooth base with a little fibrin attached and nonelevated, punched-out margins, in contrast to gastric carcinoma, which often has an irregular necrotic base and firm, raised margins. Despite these characteristic findings, the distinction between gastric peptic ulcer and ulcerated carcinoma must be established by biopsy. In contrast to carcinoma, peptic ulcer will usually heal with conservative management.
12
Q
  1. A 20-year-old man presents with severe right lower quadrant abdominal pain, nausea, and anorexia. He states that the abdominal pain started around his umbilicus and has now migrated to the right lower quadrant of his abdomen. Physical examination reveals exquisite tenderness at McBurney point (the point one-third of the distance along the line from the right anterior superior iliac spine to the umbilicus). This patient is diagnosed with acute appendicitis. Which of the following is the treatment for this condition? (A) Antibiotics only, because the appendix is crucial for survival (B) Surgical resection of the appendix, because appendicitis can lead to appendiceal cancer (C) Surgical resection of the appendix, because appendicitis can lead to perforation or abscess (D) “Watch-and-wait” approach over days to see if inflammation subsides
A
  1. T he answer is C. The inflamed appendix in acute appendicitis should be surgically removed because of possible devastating complications of perforation or abscess.
13
Q
  1. In a routine colonoscopy, a 76-year-old man is found to have a lesion similar to that shown in the illustration. The lesion shown is a classic example of which of the following? (A) Hamartoma (B) Invasive adenocarcinoma (C) Peutz-Jeghers polyp (D) Tubular adenoma (E) Villous adenom
A
  1. T he answer is D. The illustration shows a tubular adenoma, which is the most common form of adenomatous polyp. These lesions can be single or multiple, or they can occur as components of various multiple polyposis syndromes. Notable among these syndromes are Gardner (associated with osteomas and soft tissue tumors), Turcot (associated with central nervous system tumors), and FAP. All of the foregoing are associated with an increased incidence of colon malignancy. In contrast, the Peutz-Jeghers polyp is a nonneoplastic hamartomatous lesion. Even though the polyp itself does not transform into colon cancer, the Peutz-Jeghers syndrome is associated with an increased incidence of colon cancer and malignancies elsewhere.
14
Q
  1. A 35-year-old man undergoes gastrectomy for gastric carcinoma. Gross examination of the resected stomach reveals diffuse thickening without a discrete mass lesion. Microscopic exam shows an infiltration of signet-ring cells dispersed singly. Family history reveals that his father had a similar cancer at a young age. What gene is most likely to be mutated in this patient and his father? (A) APC (B) CDH1 (C) MSH2 (D) PMS2 (E) p53
A
  1. T he answer is B. This patient most likely has a mutation in CDH1, the gene encoding the cellular adhesion protein E-cadherin. E-cadherin mutations account for a significant proportion of familial gastric cancers and are also implicated in lobular carcinoma of the breast. CDH1-mutated gastric adenocarcinomas typically show signet-ring morphology with diffuse infiltration resulting in a “linitis plastic” (“leather bottle”) gross appearance. APC is mutated in the majority of colorectal adenocarcinomas. MSH2 and PMS2 are both mismatch repair genes that may be mutated in Lynch syndrome. Although Lynch syndrome patients are at increased risk for gastric adenocarcinoma, their tumors are typically conventional, rather than signet-ring, in morphology. p53 is mutated in a wide variety of sporadic cancers and shows germline mutations in Li Fraumeni syndrome.
15
Q
  1. A 60-year-old woman develops a rightsided colonic adenocarcinoma. She has a history of alleged colonoscopies; however, review of images from her most recent colonoscopy reveals a prominent fold in the region that subsequently developed cancer. Biopsy from this area would have most likely revealed which of the following? (A) Tubular adenoma (B) Hyperplastic polyp (C) Peutz-Jeghers polyp (D) Sessile serrated adenoma (E) Inflammatory polyp
A
  1. T he answer is D. Sessile serrated adenomas can be very subtle and are easily missed on endoscopy. Under the microscope, they closely resemble hyperplastic polyps but show more complicated glands, often with “boot-shaped” configurations at the gland bases. Unlike tubular adenomas, they lack the overtly dysplastic epithelium and often do not form polypoid lesions. Hyperplastic polyps, Peutz-Jeghers polyps, and inflammatory polyps are not malignant precursors.
16
Q
  1. A 45-year-old man complains of “heartburn” and burning epigastric pain, relieved by antacids and triggered by eating spicy or acidic foods or by assuming a recumbent position. The patient smokes two packs of cigarettes a day and consumes several alcoholic drinks each evening. Which of the following is the usual cause of this patient’s condition? A) Columnar intestinal metaplasia of esophageal squamous epithelium (B) Excessive acid production in the stomach (C) Excessive NSAID use (D) H. pylori infection (E) Hiatal hernia and incompetent lower esophageal sphincter
A
  1. T he answer is E. This is a classic case of GERD, which is caused by reflux of gastric acid contents into the lower esophagus. GERD manifests as burning epigastric pain on eating spicy foods or on lying recumbent. The pain is usually relieved by antacids. GERD is most commonly associated with hiatal hernia and an incompetent lower esophageal sphincter, as well as with excessive use of alcohol or tobacco, increased gastric volume, pregnancy, and scleroderma. Barrett esophagus, or columnar intestinal metaplasia of the epithelium of the distal esophagus, is a complication of long-standing GERD.
17
Q

Large bowel biopsy, Image with tubules w/ necrosis. Most likely? A. SCC B. Fibrosarcoma C. Adenocarcinoma D. Small cell carcinoma E. Goblet cell carcinoma

A

C. Adenocarcinoma Gland like malignancy= adenocarcinoma

18
Q
  1. 52 y.o male presents w/ weight loss, lethargy and dry cough. Coughed up blood, history of hypertension that is well controlled. BMI is 30 and is on statin drug. Image shoud hilar mass in right lung. Biopsies show well to moderately differenated malignant neoplasm in infiltrating mosaic arrays, keratin pearls discerable a. adenocarcinoma b. Squamous cell carcinoma c. Papillary carcinoma d. Synovial sarcoma e. Follicular lymphoma
A

Answer: Squamous cell carcinoma Most central cancers are squamous cell carcioma Keratin pearls so squamous Squamous cancers, look like mosaics, producing keratin Adenocarcinoma= is more common?? But not related to smoking****

19
Q
  1. Drug abuse, Cetnrilobular necrosis of liver → liver failure. Danger for chronic alcholics. Early treatment with N-acetylcystein may limit liver damage. Which following a. Cocaine b. Oxycodone c. Heroin d. Acetaminophin e. Methamphetamine
A

Acetaminophin

20
Q
  1. A 68 y.o asian America man is killed in a hit and run accident while on his way home from his job at the local plastics factor where he was exposed to vinyl chloride. As part of the local law regarding motor vehicle homicides he undergoes a forensic autopsy. Which malignancy associated with his occupation is he most at risk for? a. Squamous cell carcinoma of the lung b. Angiosarcoma of the liver c. Brain tumor d. A tumor positive for desmin e. Neuroblastoma
A

Answer: B. ANgiosarcoma of the liver **Pay attention to chart which has exposures and related conditions** Squamous cell carcinoma of the lungs= smoking Pay attention to cancer ones

21
Q
  1. A 65 y.o obese Caucasian female present with complains of increased urination. She has long history of T2DM and is not very compliant. Her hemoglobin A1C is >8. Which of the following morphologic pattern of acute inflammation is most likely to involve the lower extremity of this patient? a. Serous b. Granulomatous c. Abscess d. Ulcer e. Fibrinous
A

Answer: D. Ulcer Serious= exudate of cell-poor fluid into cell space or body cavities Granulomatous= chronic process foreign body or immune mediated Abscess= localized collection of purulent material due to bacteria Ulcer= sloughing or shedding of inflamed necrotic tissue mouth, lower extremity Fibrinous= characteristic in body cavities i/e pericardium Necrotizing granulomatous inflammation= tuberculosis (tubercle)

22
Q
  1. A 9 y.o is admitted to the hospitial with fever and cough. Past medical history includes meconium ileus at birth and multiple episodes of pneumonia, the first at age 2. Chest radiograph is abnormal and shows bilateral lobar infiltrates (image). Cough is productive of thick tenacious mucus. Cultures of sputum reveal an alginate producing strain of pseudomonas aeruginosa. Most like diagnosis? a. Bronchopulmonary dysplasia b. Marfan syndrome c. Cystic fibrosis d. Pulmonary edema e. Metastic neuroblastoma
A

Answer: Cystic fibrosis

23
Q
  1. 22 y.o Hispanic female, Texas, Difficulty swallowing, SOB, distal esophageal dilation. Indicate heart is symmetrically dilated. CBC w/ mild eosinophilia. Kissing bug a. Pontiac fever b. Chagas disease c. Kala azar d. River blindness e. Rhabdoviral syndrome
A

Answer: Chagas disease – Trypanosoma cruzi • Pontiac fever: water, multiple people • Kala azar: distended, massive hepatosplenomegaly, sand flies • River blindness: rapid keratinitis, black fly • Rhabdoviral syndrome: babies, foaming at mouth, Rabies

24
Q
  1. Georgia, 7 y.o forrest barefoot, superficial lesion, itch, cough, diarrhea. Infiltrates bilateral, Iron deficiency anemia and eosinphilia a. Round worm b. Pin worm c. Microfilariae d. Hookworm e. Cutaneous larval migrans
A

Answer: Hookworm • Necator & stronglovirdes: penetrate, hook worm • Cutaneous larval migrans: ancyclostoma, dog hook worm • Larval currans: stronglyoides (moves fast)

25
Q
  1. In Haiti, a 9 y.o w/ severe watery diarrhea. Many dying due to epidemic dystentery. There are flecks of mucopurulent material in his stool. His eyes are sunken and he is listless/ disoriented. Fecal cultures aer reported as normal GI flora. However, dark field microscopy shows a motile organism a. Salmonella typhi b. Vibrio c. Treponemal organism d. Campylobacter e. EHEC
A

Answer: Virbrio • Virbrio: watery diarrhea, cholleria, containmanted water (rice water stool) • Salmonella: bloody, acquired orally • EHEC: bloody • Campylobacter: how is it acquired? • EHEC: hemoharrigc form of ECOLI (HUS, thrombocytopenia, uremia, and …..)

26
Q

1 A 23-year-old primigravida gives birth at term to a boy infant. Ultrasound examination before delivery showed polyhydramnios. A single umbilical artery is seen at the time of birth. The infant vomits all feedings, and then develops a fever and difficulty with respirations within 2 days. A radiograph shows both lungs and the heart are of normal size, but there are pulmonary infiltrates and no stomach bubble. What is the most likely diagnosis? A Achalasia B Diaphragmatic hernia C Esophageal atresia D Hiatal hernia E Pyloric stenosis F Zenker diverticulum

A

C An esophageal atresia is often combined with a fistula between the esophagus and trachea. Gastrointestinal obstruction in utero can lead to polyhydramnios. The presence of a single umbilical artery suggests additional anomalies are present. Vomiting in an infant risks aspiration with development of pneumonia. Achalasia is incomplete relaxation of the lower esophageal sphincter and is usually not manifested at birth. Absence of a diaphragmatic leaf, usually on the left, results in herniation of abdominal contents into the chest and functional gastrointestinal obstruction, but in this case normal-sized lungs suggest no herniated contents were present. A hiatal hernia from widened diaphragmatic muscular crura predisposes to gastroesophageal reflux, and obstruction is not a typical complication. Pyloric stenosis is a cause for gastric outlet obstruction in an infant, but the onset is usually in the second or third week of life. A pharyngoesophageal (Zenker) diverticulum above

27
Q

53 y.o presents w/ abdominal pain and sclerolicterus (yellow eyes). His HBeAg is positive, anti-HbsAg negative and IgG anti-HbcAg psotive. Hepatic imaging indicates multinodularity and malignancy. Most likely? a. Cholangiocarcinoma b. Hepatocellular carcinoma c. Neuruoblastoma d. Hepatic adenoma e. Metastic cancer to liver, NOS

A

Answer: Hepatocellular carcinoma • Hepititis B • Not Hepatitis C: puts at risk for Hepatocellular carcinoma, long term

28
Q
  1. Boy scout troup to burger shop. Vomiting, running high fever, severe abdominal pain. Guarding. Hemorrhage hemolytic syndrome a. Campylobacter sp. b. Staphylococcus aureus c. Bacillus anthracis d. Escherichia coli e. Salmonella sp.
A

Answer: E Coli • Campylobacter sp: blood diarhhea, arthritis, Gion berra, can have affiliation w/ hemolytic… • Staph aureus • Bacillus anthraci: inhalation, GI, cutanous (person would be dead) • Travels something is not invasive → watery diarrhea • E coli: Group B or C weapon of mass destruction

29
Q
  1. Severe abdominal pain, 99.9 F, tachycardic. Discomfort lower quadrants on left. Rebound tenderness. Diarrhea, loose, dark stool. Treated for asthma when teenager has frequent dry cough. Image indicates severe bowel obstructions- right ascending colon and sigmoid. Most likely? a. Fungal infection b. Yersinia enterocolitica c. Colon Carcinoma d. Parasite infestation e. Bacterial/ Viral infection
A

Answer: Yersinia enterocolitica • Rebound tenderness: appendicitis, peritonitis • Diarrhea w/ dark stool= blood, upper GI (has had time to work its way through) • Bowel obstructions: look at region these are in • Most common presentations of fungal infections: skin (so probably not fungal) • Colon cancer: possible but imaging and symptoms wise not likely • Parasite infestation: obstruction, could be possible • PICK UP ON: Yersinia enterocolitica → pseudo appendicitis can also travel to lymph nodes and do ….

30
Q
  1. Severe abdominal pain, 99.9 F, tachycardic. Discomfort lower quadrants on left. Rebound tenderness. Diarrhea, loose, dark stool. Treated for asthma when teenager has frequent dry cough. Image indicates severe bowel obstructions- right ascending colon and sigmoid. Most likely? His CBC indicates significant iron deficiency anemia and eosinophilia. Admit to hospitial. Most likely? a. Fungal infection b. Yersinia enterocolitica c. Colon carcinoma d. Parasite infection e. Bacterial/ Viral infection
A

Answer: Parasite infection • HM: iron deficiency anemia and eosinophils • Iron deficiency anemia= necator** • Tape worm that causes anemia= B12, Delatum?? Fish worm

31
Q

3 A 23-year-old woman, G2, P1, gave birth at term to a boy of normal weight and length following an uncomplicated pregnancy. The infant initially did well, but at 6 weeks, he began feeding poorly for 1 week, and his mother noticed that much of the milk he ingested was forcefully vomited within 1 hour. Now, on physical examination, the infant is afebrile, and there are no external anomalies. A midabdominal mass is palpable. Bowel sounds are active. The medical history indicates that both the mother and her first child had the same illness during infancy. Which of the following conditions is most likely to explain these findings? A Annular pancreas B Diaphragmatic hernia C Duodenal atresia D Pyloric stenosis E Tracheoesophageal fistula

A

3 D The infant’s condition occurred several weeks after birth because of hypertrophy of pyloric smooth muscle. Pyloric stenosis has features of multifactorial inheritance with a “threshold of liability,” above which the disease is manifested when more genetic risks are present, such as family history and twin gestation. The incidence in males is 1 in 200 and in females is 1 in 1000, reflecting the fact that more risks must be present in females for the disease to occur. Annular pancreas is a rare anomaly that can also cause obstruction of the duodenum, and has variable age of onset, but a palpable mass would not be expected. Tracheoesophageal fistula, diaphragmatic hernia, and duodenal atresia are serious conditions that are manifested at birth and are often associated with multiple anomalies. Pyloric stenosis is an isolated condition that typically occurs without other anomalies.

32
Q

A 24-year-old man has developed abdominal pain and increasing fatigue over the past 6 months. On physical examination, he is afebrile and appears pale. On palpation, there is mild pain in the right lower quadrant of the abdomen. There are no masses, and bowel sounds are active. Laboratory studies show hemoglobin, 8.9 g/dL; hematocrit, 26.7%; MCV, 74 μm3; platelet count, 255,000/mm3; and WBC count, 7780/ mm3. His stool is positive for occult blood. Upper gastrointestinal endoscopy and colonoscopy showed no lesions. One month later, he continues to experience the same abdominal pain. Which of the following is most likely to cause this patient’s illness? A Acute appendicitis B Angiodysplasia C Celiac disease D Diverticulosis E Giardia lamblia infection F Meckel diverticulum

A

F About 2% of individuals have a Meckel diverticulum, an embryologic remnant of the omphalomesenteric duct, but only a small subset of these individuals have ectopic gastric mucosa within it, which causes intestinal ulceration. The symptoms may mimic acute appendicitis, but appendicitis should not last for 1 month or cause significant blood loss. Angiodysplasia may be difficult to detect, and it is almost always seen in patients older than 70 years, but can cause significant blood loss. Celiac disease can occur in young individuals, but it does not produce significant hemorrhage. Diverticulosis can be associated with hemorrhage, but the diverticula are almost always in the colon of older persons. Giardiasis produces a self-limited, watery diarrhea without hemorrhage.

33
Q

4 A 24-year-old woman gives birth to term infant after an uncomplicated pregnancy. Apgar scores are 9 and 10 at 1 and 5 minutes after birth. The infant’s length and weight are at the 55th percentile. There is no significant passage of meconium. Three days after birth, the infant vomits all oral feedings. On physical examination, the infant is afebrile, but the abdomen is distended and tender, and bowel sounds are reduced. An abdominal ultrasound scan shows marked colonic dilation above a narrow segment in the distal sigmoid region. A biopsy specimen from the narrowed region shows an absence of ganglion cells in the muscle wall and submucosa. Which of the following is most likely to produce these findings? A Colonic atresia B Hirschsprung disease C Intussusception D Necrotizing enterocolitis E Trisomy 21 F Volvulus

A

4 B In Hirschsprung disease, seen in 1 in 5000 live births, the aganglionic segment (either a short or long segment) of the bowel wall produces a functional obstruction with proximal distention. Most familial cases and some sporadic cases have RET gene mutations affecting neural crest cell migration. Atresias are congenitally narrowed segments of bowel (usually the small intestine) that occur with other anomalies. Patients with trisomy 21 may have intestinal (usually duodenal) atresias. Complete absence of the colonic lumen at a point of atresia is a rare congenital anomaly and is not associated with loss of ganglion cells. Intussusception also is a cause of bowel obstruction in infants, but it is not caused by an aganglionic segment of bowel. Necrotizing enterocolitis is a complication of prematurity. Volvulus is a form of mechanical obstruction that occurs from twisting of the small bowel on the mesentery or twisting

34
Q

5 A 3-year-old child has attained enough mobility, curiosity, and dexterity to explore places in the home that should not be accessed. The child finds a bottle with a liquid under the kitchen sink, and he drinks it. Within minutes he has chest pain. His mother takes him to the emergency department, and brings the bottle. Analysis of the residual contents reveals a pH of 12. Which of the following complications is most likely to occur following this injury? A Pharyngeal diverticulum B Esophageal stenosis C Gastric lymphoma D Duodenal ulceration E Megacolon

A

5 B Caustic alkaline solutions tend to damage the esophagus, and may not even get as far as the stomach. If the esophagus is perforated, a severe mediastinitis may occur. The inflammation can resolve with scarring and stenosis, and that tends to affect swallowing of solids more than liquids, typical for mechanical obstruction. A pharyngeal Zenker diverticulum occurs at a point of weakness in the hypopharynx, most often between the inferior constrictor muscle and cricopharyngeus muscle; it is a pulsion diverticulum from motility problems. Gastric lymphomas may be related to Helicobacter pylori infection (MALTomas) and to immune dysregulation. Duodenal ulcerations are predominantly related to H. pylori infection. Megacolon results from marked colonic inflammation or motor disturbances, and swallowed substances are not likely to reach the colon unaltered.

35
Q

6 A 22-year-old woman has had multiple episodes of aspiration of food associated with difficulty swallowing during the past year. On auscultation of her chest, crackles are heard at the base of the right lung. A barium swallow shows marked esophageal dilation above the level of the lower esophageal sphincter. A biopsy specimen from the lower esophagus shows an absence of the myenteric ganglia. What is the most likely diagnosis? A Achalasia B Barrett esophagus C Plummer-Vinson syndrome D Sliding hiatal hernia E Systemic sclerosis

A

6 A In achalasia, there is incomplete relaxation of the lower esophageal sphincter with lack of peristalsis. Most cases are “primary” or of unknown origin. They may be caused by degenerative changes in neural innervation; the myenteric ganglia are usually absent from the body of the esophagus. There is a long-term risk of development of squamous cell carcinoma. In Barrett esophagus, there is columnar epithelial metaplasia, but the myenteric plexuses remain intact. Reflux esophagitis may be associated with hiatal hernia, but myenteric ganglia remain intact. Plummer-Vinson syndrome is a rare condition caused by iron deficiency anemia; it is accompanied by an upper esophageal web. Systemic sclerosis (scleroderma) is marked by fibrosis with stricture.

36
Q

7 A 24-year-old woman living in eastern Bolivia has had increasing difficulty with swallowing both liquids and solids for the past year. She has substernal discomfort from a feeling that foods “get stuck” going down. On examination her BMI is 18. A barium swallow radiologically shows marked esophageal dilation. An endoscopic biopsy is obtained and microscopically shows reduced ganglion cells in myenteric plexus along with lymphocytic infiltration. Which of the following organisms is most likely infecting this woman? A Bordetella pertussis B Candida albicans C Corynebacterium diphtheriae D Herpes simplex virus E Trypanosoma cruzi

A

7 E Chronic Chagas disease can lead to damage to not only myocardium but also tubular structures of the GI tract, especially the esophagus with secondary achalasia. The organisms are hard to find microscopically, but they elicit the inflammatory response that damages neurons to produce the motility problems. Pertussis is whooping cough, typically a childhood disease affecting the upper airways. Candidiasis tends to produce surface plaques with minimal erosion in immunocompromised persons. Diphtheria is most often a childhood disease of upper airways, and there can be toxin-mediated systemic disease, including myocarditis, but there is no chronic infection. Herpetic ulcers are sharply demarcated, and infection is most often found in immunocompromised persons.

37
Q

8 A 53-year-old man consumes a very large meal, washed down with considerable alcohol. The ensuing discomfort prompts him to take an emetic, but soon afterward he develops lower chest pain. Physical examination reveals crepitus in subcutaneous tissue over his chest along with tachycardia and tachypnea. Which of the following abnormalities of the esophagus is most likely present in this man? A Stricture B Achalasia C Ectopia D Rupture E Varices

A

8 D Grand Admiral Baron Jan Gerrit van Wassenaer was attended by Dr. Herman Boerhaave in 1724, who then described esophageal rupture. Boerhaave syndrome may follow forceful vomiting, or may occur as a complication of instrumentation. Dissection of air from the rupture extends into soft tissue, producing the subcutaneous emphysema. There is no serosal barrier above the diaphragm, so esophageal contents spill into the chest cavity, producing marked mediastinitis that is hard to treat. A stricture is likely to occur with long-standing inflammation or from the fibrosis associated with systemic sclerosis (scleroderma). Achalasia is a functional obstruction from failure of inhibitory neurons that relax the lower esophageal sphincter. Ectopia refers to tissue that is out of place, most often gastric mucosa that is in the esophagus, which can lead to esophagitis. Varices present a risk for marked bleeding.

38
Q

9 A 30-year-old man has sudden onset of hematemesis after a weekend in which he consumed large amounts of alcohol. The bleeding stops, but he has another episode under similar circumstances 1 month later. Upper gastroesophageal endoscopy shows longitudinal tears at the gastroesophageal junction. What is the most likely mechanism to cause his hematemesis? A Absent myenteric ganglia B Autoimmune inflammation C Herpes simplex virus infection D Portal hypertension E Vomiting F Widened diaphragmatic crura

A

9 E Mallory-Weiss syndrome with esophageal tears results from severe vomiting. Most cases occur in the context of alcohol abuse. The bleeding is usually not as life-threatening as varices. Absent myenteric ganglia occur with achalasia. Autoimmunity underlies scleroderma with fibrosis and esophageal obstruction, but there is typically no bleeding. Herpes simplex virus infection causes ulcerations that are usually superficial and cause pain, but do not bleed significantly. Portal hypertension leads to dilation of esophageal submucosal veins, which can bleed profusely; in this case, the patient’s age argues against the presence of cirrhosis from alcohol abuse. Widened diaphragmatic crura are present with hiatal hernia that predisposes to gastroesophageal reflux, and this is not associated with alcohol abuse.

39
Q

10 A 16-year-old boy who is receiving chemotherapy for acute lymphoblastic leukemia has had pain for 1 week when he swallows food. Physical examination shows no abnormal findings. Upper gastrointestinal endoscopy shows 0.5- to 0.8-cm mucosal ulcers in the region of the mid to lower esophagus. The shallow ulcers are round and sharply demarcated, and have an erythematous base. Which of the following is most likely to produce these findings? A Aphthous ulcerations B Reflux esophagitis C Herpes simplex esophagitis D Gastroesophageal reflux disease E Mallory-Weiss syndrome

A

10 C The “punched-out” ulcers described result from rupture of the herpetic vesicles. Herpesvirus and Candida infections typically occur in immunocompromised patients, and both can involve the esophagus. Aphthous ulcers (canker sores) also can be found in immunocompromised patients, but these shallow ulcers occur most frequently in the oral cavity. Candidiasis has the gross appearance of tan-toyellow plaques. Gastroesophageal reflux disease (GERD) can produce acute and chronic inflammation with some erosion, although typically not in a sharply demarcated pattern; GERD has no relationship with immune status. Mallory-Weiss syndrome results from mucosal tears of the esophagus, and laceration of the esophagus can occur with severe vomiting and retching.

40
Q

11 A 44-year-old woman has had increasing difficulty swallowing liquids and solids for the past 6 months. On physical examination, her fingers have reduced mobility because of taut, nondeforming skin. A barium swallow shows marked dilation of the esophagus with “beaking” in the distal portion, where there is marked luminal narrowing. A biopsy specimen from the lower esophagus shows prominent submucosal fibrosis with little inflammation. Which of the following is most likely to produce these findings? A Barrett esophagus B Hiatal hernia C Iron deficiency D Portal hypertension E Systemic sclerosis

A

11 E Esophageal dysmotility is the E in CREST, a mnemonic that details the key findings with the limited form of systemic sclerosis (scleroderma): C = calcinosis; R = Raynaud phenomenon; E = esophageal dysmotility; S = sclerodactyly; T = telangiectasias. Although scleroderma is an autoimmune disorder that often includes formation of anticentromere antibodies, little inflammation is seen by the time the patient seeks clinical attention. There is increased collagen deposition in gastrointestinal submucosa and muscularis. Fibrosis may affect any part of the gastrointestinal tract, but the esophagus is the site most often involved. For a diagnosis of Barrett esophagus, columnar metaplasia must be seen histologically, and there is often a history of gastroesophageal reflux disease. Hiatal hernia is frequently diagnosed in individuals with reflux esophagitis and can lead to inflammation, ulceration, and bleeding, but formation of a stricture is uncommon. An upper esophageal web associated with iron deficiency anemia might produce difficulty in swallowing, but this condition is rare. Portal hypertension gives rise to esophageal varices, not fibrosis.

41
Q

12 A 57-year-old woman has had burning epigastric pain after meals for more than 1 year. Physical examination shows no abnormal findings. Upper gastrointestinal endoscopy shows an erythematous patch in the lower esophageal mucosa. A biopsy specimen shows basal zone squamous epithelial hyperplasia, elongation of lamina propria papillae, and scattered intraepithelial neutrophils with some eosinophils. Which of the following is the most likely diagnosis? A Barrett esophagus B Esophageal varices C Iron deficiency D Reflux esophagitis E Systemic sclerosis

A

12 D Her ongoing inflammatory process results from reflux of acid gastric contents into the lower esophagus. Gastroesophageal reflux disease (GERD) is a common problem that stems from a variety of causes, including sliding hiatal hernia, decreased tone of the lower esophageal sphincter, and delayed gastric emptying. Patients may have a history of heartburn after eating. Barrett esophagus is a complication of long-standing GERD and is characterized by columnar metaplasia of the squamous epithelium that normally lines the esophagus. There may be inflammation and mucosal ulceration overlying varices, but this condition usually does not have heartburn as the major feature. Esophageal varices from portal hypertension can lead to marked hematemesis. A rare complication of iron deficiency is the appearance of an upper esophageal web (Plummer-Vinson syndrome). Progressive fibrosis with stenosis is found in scleroderma

42
Q

13 A 51-year-old man has sudden onset of massive emesis of bright red blood. On physical examination, his temperature is 36.9° C, pulse is 103/min, respirations are 23/min, and blood pressure is 85/50 mm Hg. His spleen tip is palpable. Laboratory studies show a hematocrit of 21%. The serologic test result for HBsAg is positive. He has had no prior episodes of hematemesis. The hematemesis is most likely to be a consequence of which of the following? A Barrett esophagus B Candida albicans infection C Esophageal varices D Reflux esophagitis E Squamous cell carcinoma F Zenker diverticulum

A

13 C Variceal bleeding is a common complication of hepatic cirrhosis, which can be an outcome of chronic hepatitis B infection. Portal hypertension leads to dilated submucosal esophageal veins that can erode and bleed profusely. Barrett esophagus is a columnar metaplasia that results from gastroesophageal reflux disease (GERD). Bleeding is not a key feature of this disease. Esophageal candidiasis may be seen in immunocompromised patients, but it most often produces raised mucosal plaques and is rarely invasive. GERD may produce acute and chronic inflammation and, rarely, massive hemorrhage. Esophageal carcinomas may bleed, but not enough to cause massive hematemesis. A Zenker diverticulum is located in the upper esophagus and results from cricopharyngeal motor dysfunction; it presents a risk for aspiration, but not for hematemesis

43
Q

14 A 55-year-old man has had increasing difficulty swallowing during the past 6 months. There are no significant findings on physical examination. Upper gastrointestinal endoscopy shows areas of erythematous mucosa 3 cm above the Z-line. A biopsy specimen from the lower esophagus has changes in the mucosal epithelium illustrated in the figure. Which of the following complications is most likely to occur as a consequence of this patient’s condition? A Achalasia B Adenocarcinoma C Diverticular formation D Lacerations (Mallory-Weiss syndrome) E Squamous cell carcinoma

A

14 B The biopsy specimen shows residual ulcerated squamous epithelium along with columnar metaplasia andfocal dysplasia, typical of Barrett esophagus. Patients with a focus of Barrett esophagus have a higher risk of developing adenocarcinoma than the general population, particularly when high-grade dysplasia is present. Achalasia refers to the failure of the lower esophageal sphincter to relax, which gives rise to dilation of the proximal portion of the esophagus. An epiphrenic diverticulum in the lower esophagus is not associated with Barrett mucosa, but arises from increased intraluminal pressure against lower esophageal sphincter obstruction. Mallory-Weiss syndrome is associated with vertical lacerations in the esophagus that may occur with severe vomiting and retching. Squamous cell carcinomas occur in the midesophagus, but they do not arise in association with Barrett esophagus. Instead, they are linked to smoking and alcohol consumption.

44
Q

15 A 68-year-old man from Birmingham, England, has had “heartburn” and substernal pain after meals for 25 years. For the past year, he has had increased pain with difficulty swallowing both liquids and solids. On physical examination, there are no remarkable findings. Upper gastrointestinal endoscopy shows an ulcerated lower esophageal mass that nearly occludes the lumen of the esophagus. A biopsy specimen of this mass is most likely to show which of the following neoplasms? A Adenocarcinoma B Carcinoid tumor C Leiomyosarcoma D Non-Hodgkin lymphoma E Squamous cell carcinoma

A

15 A Adenocarcinomas of the esophagus are typically located in the lower esophagus, where Barrett esophagus develops at the site of long-standing gastroesophageal reflux disease. Barrett esophagus is associated with an increased risk of developing adenocarcinoma, particularly when high-grade dysplasia is present. Columnar metaplasia may progress to dysplasia, then adenocarcinoma. Carcinoid tumors occur in different parts of the gut, including the appendix, ileum, rectum, stomach, and colon. Leiomyosarcoma of the esophagus is rare and is unrelated to a history of heartburn. Malignant lymphomas of the gastrointestinal tract do not commonly occur in the esophagus and are not related to reflux esophagitis. Squamous cell carcinomas of the esophagus are most often associated with a history of chronic alcoholism and smoking.

45
Q

16 A 73-year-old man with a history of chronic alcoholism has had increasing difficulty swallowing and has noticed a 3-kg weight loss over the past 2 months. On physical examination, there are no remarkable findings. Upper gastrointestinal endoscopy shows a 3-cm ulcerative mass in the midesophagus that partially occludes the esophageal lumen. Esophagectomy is performed; the gross appearance of the lesion is shown in the figure. Which of the following is most likely to be seen on microscopic section of this mass? A Adenocarcinoma B Dense collagenous scar C Dilated vascular channels D Multinucleated cells with intranuclear inclusions E Squamous cell carcinoma

A

16 E This large, ulcerated lesion with heaped-up margins is a malignant tumor of the esophageal mucosa. There are two main histologic types of esophageal carcinomas—squamous cell carcinoma and adenocarcinoma—with distinct risk factors. Smoking and alcoholism are the primary risk factors for esophageal squamous cell carcinoma in the Western world. Adenocarcinoma is most likely to arise in the lower third of the esophagus and to be associated with Barrett esophagus. Chronic inflammation may lead to stricture and not to a localized mass. Dilated veins occur in esophageal varices; they do not produce an ulcerated mass. A dense, collagenous scar of the mid esophagus is uncommon, but it may occur after injury from ingestion of a caustic liquid. Intranuclear inclusions suggest infection with herpes simplex virus or cytomegalovirus, both of which are more likely to produce ulceration without a mass; both occur in immunocompromised patients.

46
Q

17 A 66-year-old man living in Tehran, Iran, has been bothered by difficulty swallowing for the past year. He is now consuming liquid food. Yesterday he regurgitated food stained with blood. On esophagoscopy, there is an ulcerated obstructing lesion 20 cm from the lips. Biopsies are taken and on microscopy show infiltrating nests of keratinized cells with distinct cell borders and hyperchromatic, angulated nuclei. Which of the following is the most likely risk factor for his disease? A Genetic susceptibility B Autoimmunity C Diet D Infection E Reflux

A

17 C The Turkmen population around the Caspian Sea has the highest rate of esophageal cancer on earth, and most of these are squamous cell carcinomas arising in the midesophagus. Consuming hot tea, contamination with silicates in consumed food, micronutrient deficiencies, and family history have been implicated, as well as human papillomavirus infection. There are no specific gene mutations known to be associated with esophageal carcinoma in this population. In contrast, tobacco use and alcohol consumption are linked to esophageal cancers in Europe and North America. The main autoimmune disease affecting the esophagus, systemic sclerosis (scleroderma), is not a major risk for cancer. Infectious agents such as Candida and herpes simplex virus do not carry a risk for cancer; the role for human papillomavirus in this process is not well established. Reflux esophagitis is a risk for adenocarcinomas arising in the lower third of the esophagus.

47
Q

19 A 72-year-old man takes large quantities of nonsteroidal anti-inflammatory drugs (NSAIDs) because of chronic degenerative arthritis of the hips and knees. Over the past 2 weeks, he has had epigastric pain with nausea and vomiting and an episode of hematemesis. On physical examination, there are no remarkable findings. A gastric biopsy specimen is most likely to show which of the following lesions? A Acute gastritis B Adenocarcinoma C Epithelial dysplasia D Helicobacter pylori infection E Hyperplastic polyp

A

19 A Prolonged use of nonsteroidal anti-inflammatory drugs (NSAIDs) is an important cause of acute gastritis. NSAIDs inhibit cyclooxygenase-dependent synthesis of prostaglandins E2 and I2, which stimulate nearly all defense mechanisms. Excessive alcohol consumption and smoking also are possible causes. Acute gastritis tends to be diffuse and, when severe, can lead to significant mucosal hemorrhage that is difficult to control. Epithelial dysplasia may occur at the site of chronic gastritis. It is a forerunner of gastric cancer. Infection with Helicobacter pylori is not associated with acute gastritis. Hyperplastic polyps of the stomach do not result from acute gastritis, but may arise in association with chronic gastritis. Acute gastritis does not increase the risk of gastric adenocarcinoma.

48
Q

20 A 54-year-old, previously healthy man sustained an extensive thermal burn injury involving 70% of the total body surface area of his skin. He was hospitalized in stable condition. Three weeks after the initial burn injury, he developed melanotic stools. His blood pressure dropped to 80/40 mm Hg, and his hematocrit declined to 18%. Where are gastrointestinal ulcerations most likely to be found in this man? A Colon B Duodenum C Esophagus D Ileum E Stomach

A

20 E So-called stress ulcers, also known as Curling ulcers, can occur in patients with burn injuries. The ulcers are often small (<1 cm) and shallow, never penetrating the muscularis propria, but they can bleed profusely. Similar lesions can occur after traumatic or surgical injury to the central nervous system (Cushing ulcers). Duodenal ulcers are typically peptic ulcers in individuals with Helicobacter pylori infection. Esophageal varices can cause massive hematemesis, but they occur in patients with portal hypertension, caused most commonly by cirrhosis. Metaplastic columnar epithelium at the lower end of the esophagus is present in Barrett esophagus, resulting from chronic gastroesophageal reflux disease. Ileal ulcerations and colonic ulcerations are often due to inflammatory bowel disease that can be from infections such as shigellosis, or they may be idiopathic, as in Crohn disease.

49
Q

21 A 51-year-old woman has been feeling increasingly tired for the past 7 months. There are no remarkable findings on physical examination. Laboratory studies include hemoglobin, 9.5 g/dL; hematocrit, 29.1%; MCV, 124 μm3; platelet count, 268,000/mm3; and WBC count, 8350/mm3. The reticulocyte index is low. Hypersegmented polymorphonuclear leukocytes are found on a peripheral blood smear. The serum gastrin is markedly increased. Antibodies to which of the following are most likely to be found in this patient? A Gastric H+,K+-ATPase B Gliadin C Helicobacter pylori D Intrinsic factor receptor E Tropheryma whippelli

A

21 A The high MCV is indicative of a megaloblastic anemia, most likely pernicious anemia, resulting from autoimmune atrophic gastritis. Delayed maturation of the myeloid cells leads to hypersegmentation of polymorphonuclear leukocytes. Loss of gastric parietal cells from autoimmune injury causes a deficiency of both intrinsic factor and acid. In the absence of this factor, vitamin B12 cannot be absorbed in the distal ileum. Among the various anti–parietal cell antibodies are those directed against the acid-producing proton pump enzyme H+,K+-ATPase. Antigliadin antibodies are found with celiac disease that does not affect the gastric mucosa. H. pylori causes chronic gastritis and peptic ulcer disease, but does not injure parietal cells. In pernicious anemia, no antibodies are directed against intrinsic factor receptor on ileal mucosal cells. Infection with Tropheryma whippelli causes Whipple disease, which may involve any organ, but most often affects intestines, central nervous system, and joints; malabsorption is common.

50
Q

22 A 59-year-old man has had nausea and vomiting for 5 months. He has experienced no hematemesis. On physical examination, there is no abdominal tenderness, and bowel sounds are present. Upper gastrointestinal endoscopy shows erythematous areas of mucosa with thickening of the rugal folds in the gastric antrum. The microscopic appearance of a gastric biopsy specimen with a Steiner silver stain is shown in the figure. Which of the following factors is most likely responsible for this gastric mucosal pathology? A Cysteine proteinase B Cytotoxin-associated gene A C Heat-stable enterotoxin D Shiga toxin E Verocytotoxin

A

22 B Helicobacter pylori organisms shown in the figure reside in the mucus layer above the gastric mucus and are associated with various gastric disorders, ranging from chronic gastritis with erythema and thickened rugal folds, as in this case, to peptic ulcers and to adenocarcinoma. H. pylori organisms elaborate several toxic substances that injure the epithelium. The H. pylori gene from a pathogenicity island encodes cytotoxin-associated antigen (CagA) and is present in many patients with chronic gastritis and peptic ulcers; it increases the risk for gastric cancer. Cysteine proteinases produced by Entamoeba histolytica aid in tissue invasion. Heat-stable enterotoxin is produced by strains of Escherichia coli that cause traveler’s diarrhea. Shiga toxin is elaborated by Shigella flexneri organisms, which cause a form of bacillary dysentery. Verocytotoxin produced by some E. coli strains is associated with hemolytic uremic syndrome mediated by endothelial injury.

51
Q

23 A 47-year-old woman with a lengthy history of heartburn and dyspepsia experiences sudden onset of abdominal pain. On physical examination, she has severe mid epigastric pain with guarding. Bowel sounds are reduced. An abdominal plain film radiograph shows free air under the left leaf of the diaphragm. She is immediately taken to surgery, and a perforated duodenal ulcer is repaired. Which of the following organisms is most likely to have produced these findings? A Campylobacter jejuni B Cryptosporidium parvum C Giardia lamblia D Helicobacter pylori E Salmonella typhi F Shigella flexneri G Yersinia enterocolitica

A

23 D Although they are not found in the duodenum, Helicobacter pylori organisms alter the microenvironment of the stomach, causing the stomach and duodenum to be susceptible to peptic ulcer disease. Virtually all duodenal peptic ulcers are associated with H. pylori infection. Ulceration can extend through the muscularis and result in perforation, as in this case. The other organisms listed are not related to peptic ulcer formation, but to infectious diarrheal illnesses. Salmonella typhi may produce typhoid fever with more systemic symptoms; the marked ulceration of Peyer patches may lead to perforation.

52
Q

24 A 35-year-old man has had epigastric pain for more than 1 year. The pain tends to occur 2 to 3 hours after a meal and is relieved if he takes antacids or eats more food. He has noticed a 4-kg weight gain in the past year. He does not smoke and drinks 1 glass of Johannisberg Riesling daily. The result of a urea breath test is positive, and a gastric biopsy specimen contains urease. He begins a 2-week course of antibiotics, but on day 4, he feels better and discontinues treatment. Three weeks later, the epigastric pain recurs. If he does not seek further treatment, which of the following complications is he most likely to develop? A Carcinoid syndrome B Fat malabsorption C Hematemesis D Migratory thrombophlebitis E Vitamin B12 deficiency

A

24 C The clinical symptoms in this case suggest peptic ulcer disease. In most cases, peptic ulcers are associated with Helicobacter pylori infection. These bacteria secrete urease, which can be detected by oral administration of urea 14C. After drinking the labeled urea solution, the patient blows into a tube. If H. pylori urease is present in the stomach, the urea is hydrolyzed, and labeled carbon dioxide is detected in the breath sample. In the biopsy urease test, antral biopsy specimens are placed in a gel containing urea and an indicator, and if H. pylori is present, the color changes within minutes. If not properly treated, peptic ulcers can produce many complications, including massive bleeding that can be fatal. Carcinoid tumors can occur in the stomach, but they are rare and are not related to peptic ulcer disease, which this patient has. He does not have fat malabsorption because fat absorption does not occur in the stomach. Peptic ulcers rarely progress to gastric carcinoma. The stomach has numerous arterial supplies and therefore is unlikely to be affected by focal thrombosis. Vitamin B12 deficiency can occur with autoimmune atrophic gastritis because intrinsic factor, which is required for vitamin B12 absorption, is produced in gastric parietal cells.

53
Q

25 A 52-year-old man notes nausea with abdominal discomfort after meals. On physical examination, there are no abnormal findings. Upper endoscopy is performed, and there are three ovoid nodules in the fundus and antrum ranging from 0.3 to 1.2 cm in size. They have rounded, smooth surfaces. Biopsies are taken and on microscopic examination there are irregular, cystically dilated and elongated foveolar glands. Which of the following treatment strategies is most appropriate for his gastric lesions? A Antibiotics B Chemotherapy C Corticosteroids D Multivitamins E Total gastrectomy F Vagotomy

A

25 A Gastric inflammatory/hyperplastic polyps may arise in the setting of Helicobacter pylori infection. They are the most common type of gastric polyp. They may be precursors to gastric adenocarcinomas, particular lesions larger than 1.5 cm and with high-grade dysplasia. The other listed options are not appropriate for an infectious etiology.

54
Q

26 A 49-year-old woman has a history of peptic ulcer disease for which she has been treated with proton pump inhibitors. She has had nausea with vomiting for the past 2 months. Upper GI endoscopy reveals three circumscribed, round, smooth lesions in the gastric body from 1 to 2 cm in diameter. Biopsies are taken and microscopically show the lesions to consist of irregular glands that are cystically dilated and lined by flattened parietal and chief cells. No inflammation, Helicobacter pylori, metaplasia, or dysplasia is present. What is the most likely diagnosis? A Fundic gland polyps B Gastric adenomas C Hyperplastic polyps D Hypertrophic gastropathy

A

26 A There is an association of fundic gland polyps with use of proton pump inhibitors and also with familial adenomatous polyposis (FAP); increased gastrin may drive glandular hyperplasia. Gastric adenomas are most common in the antrum, have intestinal metaplasia with dysplasia, and are precursors to adenocarcinoma; they may occur with FAP. Hyperplastic polyps are associated with chronic gastritis, often from H. pylori infection. One form of hypertrophic gastropathy is Ménétrier disease, which results from excessive secretion of transforming growth factor alpha (TGF-α) with diffuse enlargement of gastric rugae and protein-losing enteropathy

55
Q

27 A 53-year-old woman has had nausea, vomiting, and midepigastric pain for 5 months. On physical examination, there are no significant findings. An abdominal CT scan shows gastric outlet obstruction. Upper gastrointestinal endoscopy shows an ulcerated 2 × 4 cm bulky mass in the antrum at the pylorus. A urease test is positive. Which of the following neoplasms is most likely to be seen in a biopsy specimen of this mass? A Adenocarcinoma B Leiomyosarcoma C Neuroendocrine carcinoma D Non-Hodgkin lymphoma E Squamous cell carcinoma

A

27 A The most likely cause of a large mass lesion in the stomach is a gastric carcinoma, and this lesion is an adenocarcinoma, likely the intestinal type found in the antral region. Adenocarcinoma is related to Helicobacter pylori infection, with β-catenin mutation. The incidence of this type of gastric cancer has been decreasing for decades in places where food processing methods have improved. Malignant lymphomas and leiomyosarcomas are less common and tend to form bulky masses in the fundus. Neuroendocrine carcinomas are rare. Squamous cell carcinomas typically appear in the esophagus.

56
Q

28 A 67-year-old woman has experienced severe nausea, vomiting, early satiety, and a 9-kg weight loss over the past 4 months. On physical examination, she has muscle wasting. Upper gastrointestinal endoscopy shows that the entire gastric mucosa is eroded and has an erythematous, cobblestone appearance. An abdominal CT scan shows that the stomach is small and shrunken. Which of the following is most likely to be found on histologic examination of a gastric biopsy specimen? A Chronic atrophic gastritis B Primary gastric lymphoma C Gastrointestinal stromal tumor D Granulomatous inflammation E Signet ring cell adenocarcinoma

A

28 E The endoscopic and radiologic findings describe the linitis plastica (“leather bottle”) appearance of diffuse gastric carcinoma. Histologically, these carcinomas are composedof the gastric type of mucus cells that infiltrate the stomach wall diffusely. The individual tumor cells have a signet ring appearance because the cytoplasmic mucin pushes the nucleus to one side. In chronic atrophic gastritis, the rugal folds are lost, but there is no significant scarring or shrinkage. Primary gastric lymphomas are less common than adenocarcinomas; a lymphoma may be large but would not involve the stomach in a diffuse pattern. Gastrointestinal stromal tumors tend to be bulky masses. Granulomas are rare at this site.

57
Q

29 A 52-year-old man has had a 4-kg weight loss and nausea for the past 6 months. He has no vomiting or diarrhea. On physical examination, there are no remarkable findings. Upper gastrointestinal endoscopy shows a 6-cm area of irregular, pale fundic mucosa and loss of the rugal folds. A biopsy specimen shows a monomorphous infiltrate of lymphoid cells microscopically. Helicobacter pylori organisms are identified in mucus overlying adjacent mucosa. Cytogenetic analysis shows t(11;18)(q21;q21). He receives antibiotic therapy for H. pylori, and the repeat biopsy specimen shows a resolution of the infiltrate. What is the most likely diagnosis? A Autoimmune gastritis B Chronic gastritis C Crohn disease D Diffuse large B-cell lymphoma E Gastrointestinal stromal tumor F Mucosa-associated lymphoid tissue tumor

A

29 F Certain gastrointestinal lymphomas that arise from mucosa-associated lymphoid tissue (MALT) are called MALT lymphomas. Gastric lymphomas that occur in association with Helicobacter pylori infection are composed of monoclonal B cells, whose growth and proliferation depend on cytokines derived from T cells that are sensitized to H. pylori antigens. Treatment with antibiotics eliminates H. pylori and the stimulus for B-cell growth. However, lesions acquiring additional mutations, such as p53, may become more aggressive. MALT lesions can occur anywhere in the gastrointestinal tract, although they are rare in the esophagus and appendix. In H. pylori chronic gastritis, which may precede lymphoma development, there are lymphoplasmacytic mucosal infiltrates. Diffuse large B-cell lymphomas and other non-Hodgkin lymphomas that are not MALT lymphomas do not regress with antibiotic therapy. Autoimmune gastritis is a risk for development of gastric adenocarcinoma. Crohn disease is rare in the stomach and is not related to H. pylori infection. Gastrointestinal stromal tumors are uncommon; these bulky tumors may be proliferations of interstitial cells of Cajal, myenteric plexus cells that are thought to be the pacemaker of the gut.

58
Q

30 A 26-year-old man is brought to the emergency department after sustaining abdominal gunshot injuries. At laparotomy, while repairing the small intestine, the surgeon notices a 1-cm mass at the tip of the appendix. The yellow-tan submucosal mass is removed, and the microscopic appearance of the mass is shown in the figure. Immunohistochemical staining is positive for chromogranin and synaptophysin but negative for Ki-67. Which of the following is the most likely cell of origin of this lesion? A Lipoblast B Ganglion cell C Goblet cell D Neuroendocrine cell E Smooth muscle cell

A

30 D The figure shows that the cytoplasm of the tumor cell contains small, dark, round granules with a dense core (neurosecretory granules), which are characteristic of neuroendocrine cells. The gross appearance of this tumor and its location also are characteristic of carcinoid tumors. Many well-differentiated neuroendocrine tumors (carcinoids) and other small, benign bowel tumors are discovered incidentally; most are 2 cm or smaller. At this size they are unlikely to act in a malignant fashion. The other listed cell types do not have neurosecretory granules.

59
Q

31 A 55-year-old man experiences episodes of diaphoresis, dyspnea, and diarrhea for 10 months. On physical examination he has midabdominal discomfort with deep palpation, and bowel sounds are reduced. There are no abnormal findings with upper endoscopy. Abdominal CT scan shows three nodules in the liver, from 1 to 3 cm in size. Laboratory studies show a high level of serum 5-hydroxyindoleacetic acid (5-HIAA). Camera endoscopy is performed, and on review of the images, there is a midjejunal mass that partially obstructs the lumen. At laparotomy a 5-cm submucosal jejunal mass is resected, and on microscopy it is composed of nests and trabeculae of round cells with pink, granular cytoplasm. The cells of this mass are most likely related to which of the following embryologic derivatives? A Endoderm B Ectoderm C Neural crest D Notochord E Splanchnic mesoderm

A

31 C Carcinoid syndrome is uncommon. It requires a malignant carcinoid tumor (neuroendocrine carcinoma) that is sufficiently large, and likely metastatic, to produce biogenic amines and derivatives such as 5-HIAA (a metabolite of serotonin). Those tumors arising in midgut (jejunum, ileum) are more likely to be malignant. Neuroendocrine cells are scattered throughout the gastrointestinal tract mucosa and are neural crest derivatives. The bowel mucosa itself is an endodermal derivative, in which connective tissues are of mesodermal origin.

60
Q

33 A 57-year-old man from Innsbruck, Austria, goes to the emergency department because of increasing abdominal pain with distention that developed over the past 24 hours. On physical examination, there is diffuse abdominal tenderness. The abdomen is tympanitic, without a fluid wave, and bowel sounds are nearly absent. There is a well-healed, 5-cm transverse scar in the right lower quadrant of the abdomen. There is no caput medusa. A stool sample is negative for occult blood. An abdominal plain film shows dilated loops of small bowel with air-fluid levels, but there is no free air. At laparotomy, the surgeon notices a 20-cm portion of reddish black ileum that changes abruptly to pink-appearing bowel on distal and proximal margins. His medical history is significant only for an appendectomy at age 25 years. Which of the following is most likely to have produced his findings? A Adenocarcinoma of the ileum B Adhesions C Crohn disease D Indirect inguinal hernia E Intussusception F Tuberculosis G Volvulus

A

33 B The patient has acute bowel obstruction, and the findings at surgery show bowel infarction. The most common causes in developed nations are adhesions, hernias, and metastases. Adhesions are most often the result of prior surgery, as in this case, and produce “internal” hernias, where a loop of bowel becomes trapped (incarcerated), and the blood supply is compromised. Loops of bowel that become trapped in direct or indirect inguinal hernias also can infarct. When metastases are the cause, the primary site is generally known, and the cancer stage is high. Primary adenocarcinomas of the small bowel are uncommon. Crohn disease can be focal and manifest with bowel obstruction, but it is uncommon in patients of this age. Intussusception can be focal, but it is uncommon. Abdominal tuberculosis may cause circumferential stricture of the bowel, and should be considered in regions where the prevalence of tuberculosis is high. Volvulus may involve the cecal or sigmoid regions of the colon (because of their mobility). When volvulus involves the small intestine, torsion around the mesentery generally occurs, and there is extensive (not segmental) small bowel infarction.

61
Q

34 An 11-month-old, previously healthy infant has not produced a stool for 1 day. The mother notices that the infant’s abdomen is distended. On physical examination, the infant’s abdomen is very tender, and bowel sounds are nearly absent. An abdominal plain film radiograph shows no free air, but there are distended loops of small bowel with air-fluid levels. Which of the following is most likely to produce these findings? A Duodenal atresia B Hirschsprung disease C Intussusception D Meckel diverticulum E Pyloric stenosis

A

34 C The infant has signs and symptoms of acute bowel obstruction. Intussusception occurs when one small segment of small bowel becomes telescoped into the immediately distal segment. This disorder can have sudden onset in infants and may occur in the absence of any anatomic abnormality. Duodenal atresia (which typically occurs with other anomalies, particularly trisomy 21) and Hirschsprung disease (from an aganglionic colonic segment) usually manifest soon after birth. Almost all cases of Meckel diverticulum are asymptomatic, although in some cases functional gastric mucosa is present and can lead to ulceration with bleeding. Pyloric stenosis is seen much earlier in life and is characterized by projectile vomiting.

62
Q

35 A 61-year-old man has had severe abdominal pain and bloody diarrhea for the past day. On physical examination, his abdomen is diffusely tender, and bowel sounds are absent. Abdominal plain films show no free air. Laboratory studies show a normal CBC and normal levels of serum amylase, lipase, and bilirubin. His Hgb A1c is 10%. He develops shock. A year ago he had an acute myocardial infarction. Which of the following lesions is most likely to be found in this man? A Appendicitis B Cholecystitis C Pancreatitis D Intestinal infarction E Pseudomembranous colitis

A

35 D The patient’s history of myocardial infarction suggests that he had severe coronary atherosclerosis, and the elevated Hgb A1c suggests diabetes mellitus. Systemic atheromatous disease most likely involves the mesenteric vessels as well, giving rise to thrombotic occlusion of the blood vessels that perfuse the bowel. The symptoms and signs suggest infarction of the gut. Acute appendicitis rarely leads to such a catastrophic illness, unless there is perforation. (The absence of free air in the radiograph argues against perforation of any viscus.) Acute pancreatitis can be a serious abdominal emergency, but the normal levels of amylase and lipase tend to exclude it. Acute cholecystitis can produce severe abdominal pain, but bloody diarrhea and absence of bowel sounds (paralytic ileus) are unlikely. Pseudomembranous colitis develops in patients receiving broad-spectrum antibiotic therapy.

63
Q

36 A 71-year-old woman with a history of rheumatic heart disease is hospitalized with severe congestive heart failure. Four days after admission, she develops cramping lower abdominal pain. On physical examination, she is afebrile. The abdomen is distended and tympanitic, without a fluid wave, and bowel sounds are absent. A stool sample is positive for occult blood. An abdominal plain film shows no free air. Colonoscopy shows patchy areas of mucosal erythema with some overlying tan exudate in the ascending and descending colon. No polyps or masses are found. What is the most likely diagnosis? A Ischemic colitis B Mesenteric vasculitis C Shigellosis D Ulcerative colitis E Volvulus

A

36 A Hypotension with hypoperfusion from heart failure is a common cause of ischemic bowel in hospitalized patients. The ischemic changes begin in scattered areas of the mucosa and become confluent and transmural over time. This can give rise to paralytic ileus and bleeding from the affected regions of the bowel mucosa. A mesenteric vasculitis is uncommon, but could lead to bowel infarction. Shigellosis is an infectious diarrhea that causes diffuse colonic mucosal erosion with hemorrhage. Ulcerative colitis usually produces marked mucosal inflammation with necrosis, usually in a continuous distribution from the rectum upward. Volvulus is a form of mechanical obstruction caused by twisting of the small intestine on its mesentery or twisting of the cecum or sigmoid colon, resulting in compromised blood supply that can lead to infarction of the twisted segment.

64
Q

37 A 60-year-old man has had increasing fatigue for the past 8 months. On physical examination, he appears pale. On digital rectal examination, no masses are palpable, but a stool sample is positive for occult blood. Auscultation of the abdomen shows active bowel sounds, and on palpation there are no masses or areas of tenderness. Laboratory studies show hemoglobin, 8.3 g/dL; hematocrit, 24.6%; MCV, 73 μm3; platelet count, 226,000/mm3; and WBC count, 7640/mm3. Colonoscopy shows no identifiable source of the bleeding. Angiography shows a 1-cm focus of dilated and tortuous vascular channels in the mucosa and submucosa of the cecum. What is the most likely diagnosis? A Angiodysplasia B Collagenous colitis C Diverticulosis D Internal hemorrhoids E Mesenteric vein thrombosis

A

37 A Angiodysplasia refers to tortuous dilations of mucosal and submucosal vessels, seen most often in the cecum in patients older than 50 years. These lesions, although uncommon, account for 20% of cases involving significant lower intestinal bleeding. Bleeding usually is not massive, but can occur intermittently over months to years. This lesion is difficult to diagnose and is often found radiographically. The focus (or foci) of abnormal vessels can be excised. Collagenous colitis is a rare cause of a watery diarrhea that is typically not bloody. Colonic diverticulosis can be associated with hemorrhage, but the outpouchings usually are seen on colonoscopy. Hemorrhoids at the anorectal junction may account for bright red rectal bleeding, but they can be seen or palpated on rectal examination. Mesenteric venous thrombosis is rare and may result in bowel infarction with severe abdominal pain.

65
Q

38 A 21-year-old man has had increasingly voluminous, bulky, foul-smelling stools and a 7-kg weight loss for the past year. There is no history of hematemesis or melena. He has some bloating, but no abdominal pain. On physical examination, there are no palpable abdominal masses, and bowel sounds are present. Which of the following laboratory findings is most likely to be present on examination of his stool? A Entamoeba histolytica trophozoites B Giardia lamblia cysts C Increased stool fat D Occult blood E Vibrio cholerae organisms

A

38 C Fat malabsorption can occur from impaired intraluminal digestion. Smelly, bulky stools containing increased amounts of fat (steatorrhea) are characteristic. Pancreatic or biliary tract diseases are important causes of fat malabsorption. Amebiasis can produce a range of findings from a watery diarrhea to dysentery with mucus and blood in the stool. Giardiasis produces mainly a watery diarrhea. Malabsorption with steatorrhea is unlikely to be associated with bleeding. Cholera results in a massive watery diarrhea.

66
Q

39 A 34-year-old woman is bothered by a low-volume, mostly watery diarrhea associated with flatulence. The symptoms occur episodically, but they have been persistent for the past year. She has experienced a 4-kg weight loss. She has no fever, nausea, vomiting, or abdominal pain. On physical examination, there are no significant findings. A stool sample is negative for occult blood, ova, and parasites, and a stool culture yields no pathogens. An upper gastrointestinal endoscopy is performed and a biopsy specimen from the upper part of the small bowel shows severe diffuse blunting of villi and a chronic inflammatory infiltrate in the lamina propria. Which of the following serologic tests is most likely to be positive in this patient? A Anticentromere antibody B Anti–DNA topoisomerase I antibody C Antimitochondrial antibody D Antinuclear antibody E Antitransglutaminase antibody

A

39 E Characteristic serologic findings with celiac disease include positive tests for antitransglutaminase, antigliadin, and antiendomysial antibodies. This chronic disease may manifest in young adulthood but may escape diagnosis. Women are affected more than men. Celiac disease results from gluten sensitivity. Exposure to the gliadin protein in wheat, oats, barley, and rye (but not rice) results in intestinal inflammation. Gliadin sensitivity causes epithelial cells to produce IL-15, which in turn leads to accumulation of activated CD8+ T cells that bear the NK cell receptor NKG2D and damage the enterocytes expressing MIC-A. A trial of a gluten-free diet is the most logical therapeutic option. Patients usually become symptom-free, and normal histologic features of the mucosa are restored. Some patients develop dermatitis herpetiformis, and a few enteropathyassociated T-cell lymphomas. Anticentromere antibody is most specific for limited scleroderma (formerly CREST syndrome) with esophageal dysmotility. The anti–DNA topoisomerase I antibody is most specific for diffuse scleroderma, in which gastrointestinal tract involvement by submucosal fibrosis may be more extensive, and malabsorption may be present. Antimitochondrial antibody is more specific for primary biliary cirrhosis. Antinuclear antibody is present in a wide variety of autoimmune diseases, but it is not characteristic of celiac sprue.

67
Q

40 A 41-year-old woman has had diarrhea and fatigue with a 3-kg weight loss over the past 6 months. On physical examination, she is afebrile and has mild muscle wasting, but her motor strength is normal. Laboratory studies show no occult blood, ova, or parasites in the stool. A biopsy specimen from the upper jejunum is obtained, and microscopic findings are reviewed. The patient begins following a special diet with no wheat or rye grain products. The change in diet produces dramatic improvement. Which of the following microscopic features is most likely to be seen in the biopsy specimen? A Crypt abscesses and mucosal ulceration B Foamy macrophages within the lamina propria C Lymphatic obstruction D Noncaseating granulomas E Villous blunting and flattening

A

40 E The malabsorption responded to dietary treatment. She probably has celiac disease (gluten sensitivity) with histologic features including flattening of the mucosa, diffuse and severe atrophy of the villi, crypt hyperplasia, and chronic inflammation of the lamina propria. There is an increase in intraepithelial lymphocytes, both CD4+ and CD8+. Affected persons are HLA-DQ2 or HLA-DQ8 positive. Crypt abscesses are nonspecific and can be seen in inflammatory bowel disease. Lymphatic obstruction occurs in Whipple disease, and in addition, foamy macrophages accumulate in the lamina propria. The macrophages contain PAS-positive granules that under electron microscopy show an actinomycete called Tropheryma whippelli. Noncaseating granulomas are found in the intestinal wall in Crohn disease.

68
Q

41 An epidemiologic study of children with failure to thrive is undertaken in Guatemala. Some of these children with ages 1 to 3 years have repeated bouts of diarrhea, but do not improve with dietary supplements. Jejunal biopsies show blunted, atrophic villi with crypt elongation and chronic inflammatory infiltrates. What is the most likely factor contributing to recurrent diarrhea in these children? A Abetalipoproteinemia B Bacterial infection C Chloride ion channel dysfunction D Disaccharidase deficiency E NOD2 gene mutations

A

41 B Environmental enteropathy affects millions of children worldwide. Recurrent infection sets up a cycle of mucosal injury and inflammatory response that produces an appearance similar to celiac disease. There is no single infectious agent implicated, but likely there are many pathogens that cumulatively contribute to mucosal damage. Abetalipoproteinemia is a rare condition from mutations in microsomal triglyceride transfer protein that impairs enterocyte transport of lipoproteins. Cystic fibrosis results from CFTR gene mutations affecting chloride ion channels, but the resultant diarrhea is primarily from loss of pancreatic function. The most common disaccharidase deficiency is lactase deficiency, with milk intolerance. NOD2 gene mutations may contribute to Crohn disease.

69
Q

42 A 40-year-old man has episodic abdominal bloating, flatulence, and explosive diarrhea. These symptoms appear to be related to the milk shakes that he loves to consume. On physical examination, there are no remarkable findings. Laboratory studies show no increase in stool fat and no occult blood, ova, or parasites in the stool. A routine stool culture yields no pathogens. When he does not consume milk shakes or ice cream sodas, he is not symptomatic. Which of the following conditions best accounts for these findings? A Autoimmune gastritis B Celiac disease C Cholelithiasis D Cystic fibrosis E Disaccharidase deficiency

A

42 E Disaccharidase (lactase) deficiency, either congenital or acquired, is symptomatic when the lactose in milk products is not broken down into glucose and galactose by terminal digestion, resulting in an osmotic diarrhea and gas production from gut flora. Affected individuals do not always make the connection between diet and symptoms, or they do not consume enough milk products to become symptomatic. An autoimmune gastritis is most likely to result in vitamin B12 malabsorption. Celiac disease also is diet related and results from sensitivity to gluten in some grains. Cholelithiasis can cause biliary tract obstruction with malabsorption of fats and pain in the right upper quadrant of the abdomen. Cystic fibrosis affects the pancreas and mainly produces fat malabsorption

70
Q

43 A potluck lunch party is held at the office at noon. Various meats, salads, breads, and desserts that were brought in earlier that morning are served. Everyone has a good time, and most of the food is consumed. By midafternoon, the single office restroom is being used by many employees who have vomiting and acute, explosive diarrhea accompanied by abdominal cramping. Which of the following infectious agents is most likely responsible for this turn of events? A Bacillus cereus B Clostridium difficile C Escherichia coli D Salmonella enterica E Staphylococcus aureus F Vibrio parahaemolyticus

A

43 E The clinical features suggest food poisoning caused by the ingestion of a preformed enterotoxin. Staphylococcus aureus grows in food (milk products and fatty foods are favorites) and elaborates an enterotoxin that, when ingested, produces diarrhea within hours. Bacillus cereus is better known for growing on reheated fried rice; it produces an exotoxin that causes acute nausea, vomiting, and abdominal cramping. Clostridium difficile can produce a pseudomembranous colitis in patients treated with broad-spectrum antibiotics. Some strains of Escherichia coli can produce various diarrheal illnesses, but without a preformed toxin. Salmonella enterica is most often found in poultry products, but the diarrheal illnesses develop within 2 days. Vibrio parahaemolyticus is found in shellfish.

71
Q

44 A healthy 21-year-old woman develops a profuse, watery diarrhea 1 day after a meal of raw oysters. On physical examination, her temperature is 37.5° C. A stool sample is negative for occult blood. There is no abdominal distention or tenderness, and bowel sounds are present. The diarrhea subsides over the next 3 days. Which of the following organisms is most likely to produce these findings? A Cryptosporidium parvum B Entamoeba histolytica C Staphylococcus aureus D Vibrio parahaemolyticus E Yersinia enterocolitica

A

44 D Raw or poorly cooked shellfish can be the source of Vibrio parahaemolyticus, which tends to produce a milder diarrhea than Vibrio cholerae. Vibrio organisms produce a toxin that increases adenylate cyclase, leading to chloride ion secretion and osmotic diarrhea. Cryptosporidium as a cause of watery diarrhea is most often found in immunocompromised individuals. Entamoeba histolytica produces colonic mucosal invasion along with exudation and ulceration; stools contain blood and mucus. Staphylococcus aureus can produce food poisoning through elaboration of an enterotoxin that causes an explosive vomiting and diarrhea within 2 hours after ingestion. Yersinia enterocolitica is invasive and can produce extraintestinal infection.

72
Q

45 A 26-year-old man traveling to Ching Mai, Thailand, had fever, headache, and muscle pains for a day followed by watery diarrhea of 5 to 10 stools per day for 4 days. In the past day, the diarrhea has been bloody and accompanied by tenesmus. On physical examination there is diffuse abdominal pain. Microscopic examination of the stool shows numerous leukocytes and gram-negative curved rods. The diarrhea subsides, but 2 weeks later he has increasing weakness in his legs. Which of the following organisms is most likely to produce his disease? A Bacillus cereus B Campylobacter jejuni C Clostridium perfringens D Giardia lamblia E Rotavirus

A

45 B The source of Campylobacter jejuni can include contaminated water, unpasteurized milk, and poorly cooked poultry. The bloody diarrhea (dysentery) and leukocytes suggest intestinal mucosal invasion by a bacterial organism. An ascending paralysis (Guillain-Barré syndrome) may complicate some Campylobacter infections because of crossreactivity between human ganglioside GM1 and bacterial lipopolysaccharide. Bacillus cereus food poisoning tends to produce abrupt onset of vomiting. Clostridium perfringens tends to produce gas gangrene. Giardiasis produces a watery diarrhea without dysentery or extraintestinal complications. Rotavirus infections are most common in children.

73
Q

46 A 36-year-old man experiences cramping abdominal pain with fever and watery diarrhea 2 days after eating a chicken salad sandwich. Physical examination shows mild diffuse abdominal pain on palpation, but there are no masses. Bowel sounds are present. A stool sample is negative for occult blood. He recovers completely within 5 days without treatment. Which of the following infectious organisms is most likely to produce these findings? A Bacillus cereus B Entamoeba histolytica C Escherichia coli D Rotavirus E Salmonella enterica F Staphylococcus aureus G Yersinia enterocolitica

A

46 E Infection by one of several Salmonella enterica (not Typhi) causes a self-limited diarrhea. This is a form of food poisoning, typically from contaminated poultry products. Bacillus cereus growing in foods such as reheated fried rice produces an exotoxin, which, on ingestion, can produce acute onset of nausea, vomiting, and abdominal pain. Amebiasis from Entamoeba histolytica can be an invasive, exudative infection. The stools contain blood and mucus. Various diseases result from contamination with different strains of Escherichia coli, based on the characteristics of the organisms, and whether they invade or produce an enterotoxin. Poultry products are usually not contaminated with E. coli. Rotavirus is most likely to produce symptomatic watery diarrhea in children, unrelated to diet. Staphylococcus aureus causes an acute onset of abdominal pain, bloating, and diarrhea, not by directly infecting the gastrointestinal tract, but by producing an exotoxin while growing on food that is subsequently ingested. Yersinia enterocolitica is most often found in contaminated milk or pork products and may disseminate to produce lymphadenitis and further extraintestinal infection

74
Q

47 In an epidemiologic study of infections of the gastrointestinal tract, cases of patients living in Haiti from whom definitive cultures were obtained are analyzed for clinical and pathologic findings that may be useful for diagnosis. A group of patients is identified who initially had abdominal pain and diarrhea during week 1 of their illness. By week 2, these patients had splenomegaly and elevations in serum AST and ALT levels. By week 3, they were septic and had leukopenia. At autopsy, the patients who died were found to have ulceration of Peyer patches. Which of the following infectious agents is most likely to produce these findings? A Campylobacter jejuni B Clostridium perfringens C Mycobacterium bovis D Salmonella typhi E Shigella sonnei F Yersinia enterocolitica

A

47 D Typhoid fever begins as an intestinal infection, but it becomes a systemic illness. A chronic carrier state can occur in some infected individuals, with colonization of the gallbladder. Campylobacter jejuni may produce dysentery, but generally not systemic disease. Clostridium perfringens can cause gas gangrene. Mycobacterium bovis is now rare because of pasteurization of milk products; it was best known as a cause of bowel obstruction from circumferential ulceration and scarring of the small bowel. Shigellosis can produce dysentery, but the infection is generally limited to the colon. Infection with Yersinia enterocolitica can produce extraintestinal infection with lymphadenitis, but generally not dysentery.

75
Q

48 A 65-year-old woman is being treated in the hospital for pneumonia complicated by septicemia. She has required multiple antibiotics and was intubated and mechanically ventilated earlier in the course. On day 20 of hospitalization, she has abdominal distention. Bowel sounds are absent, and an abdominal radiograph shows dilated loops of small bowel suggestive of ileus. She has a low volume of bloody stool that is positive for Clostridium difficile toxin. Laboratory studies show leukocytosis and hypoalbuminemia. At laparotomy, a portion of distal ileum and cecum is resected. The gross appearance of the mucosal surface is shown in the figure. What is the most likely diagnosis? A Gas gangrene with myonecrosis B Inflammatory bowel disease C Ischemic bowel disease D Pseudomembranous enterocolitis E Toxic megacolon

A

48 D The opened colon shows pseudomembranes that are patches of fibrinopurulent debris attached to the mucosa. Pseudomembranous enterocolitis is a complication of broadspectrum antibiotic therapy, which alters gut flora to allow overgrowth of Clostridium difficile or other organisms that are capable of inflicting mucosal injury. Clostridium septicum infection can lead to myonecrosis that is most often associated with malignancy or immunosuppression. An inflammatory bowel disease does not typically produce a pronounced exudate and is not associated with C. difficile. This gross pattern also can appear from ischemic injury that is vascular or mechanical, but this patient’s history and the time course support an iatrogenic cause. An ischemic colitis resulting from mesenteric artery thrombosis could appear similar, but it is not associated with C. difficile. A dilated, thinned, toxic megacolon is an uncommon complication of ulcerative colitis.

76
Q

49 Over a holiday weekend, more than 100 adults at a resort hotel develop a diarrheal illness marked by voluminous, watery stools more than 10 times per day. They also report headache, abdominal cramping pain, and myalgias. On physical examination they have manifestations of dehydration and mild fever. Laboratory studies of stool samples show no increase in leukocytes or fat, and no RBCs. Their illness lasts just 1 to 3 days and resolves with no sequelae. Which of the following infectious agents is the most likely cause for their illness? A Cytomegalovirus B Clostridium botulinum C Norovirus D Staphylococcus aureus E Strongyloides stercoralis F Vibrio cholerae

A

49 C Norovirus outbreaks result from contamination of food or water, most often in venues where multiple persons congregate. Was it the resort pool? Noroviruses, as well as the Giardia parasite, are resistant to chlorination. Was it the buffet? Salads, shellfish, and meats are often implicated. The voluminous diarrhea suggests small intestinal involvement. The lack of leukocytes makes bacterial infection less likely. Cytomegalovirus infections are more likely in immunocompromised persons. Botulism leads to paralysis from a neurotoxin. Staphylococcal food poisoning tends to be abrupt in onset and of short duration. Strongyloidiasis tends to persist for months to years. Cholera produces life-threatening fluid loss.

77
Q

50 A 5-month-old, previously healthy infant girl in Bangladesh develops a watery diarrhea that lasts for 1 week. The infant has a mild fever during the illness, but has no abdominal pain or swelling. On physical examination, her temperature is 37.7° C. A stool sample is negative for occult blood, ova, or parasites. Her parents are told to give her plenty of fluids, and she recovers fully. Which of the following organisms is most likely to produce these findings? A Campylobacter jejuni B Cryptosporidium parvum C Escherichia coli D Listeria monocytogenes E Norwalk virus F Rotavirus G Shigella flexneri

A

50 F Rotavirus is the most common cause of viral gastroenteritis in children. It is a self-limited disease that affects mostly infants and young children, who can lose a significant amount of fluid relative to their size and can quickly become dehydrated. The death rate is less than 1%. Campylobacter jejuni is more often seen in children and adults as a food-borne cause of fever, abdominal pain, and diarrhea. Cryptosporidiosis most often causes a watery diarrhea in immunocompromised adults. Enterohemorrhagic strains of Escherichia coli can produce hemolytic uremic syndrome in young children. Listeriosis can be a congenital infection that is present along with meningitis and sepsis at birth; in infants, children, and adults, it is a food-borne or water-borne infection that tends to occur in epidemics. Norwalk virus is a common cause of diarrheal illness in adults. Shigellosis produces dysentery with bloody diarrhea.

78
Q

51 A study of children living in rural Malawi in Africa reveals a high prevalence of iron deficiency anemia. Stool samples are positive for occult blood. Pruritus of the skin of their feet as well as cough are additional findings in many of these children. Which of the following parasitic infestations is the most likely cause for these findings? A Ancylostoma duodenale B Ascaris lumbricoides C Cryptosporidium parvum D Enterobius vermiculari

A

51 A Hookworm infections may be caused by Ancylostoma duodenale (Old World) or Necator americanus (New World) or both, because the geographic distributions may overlap, particularly in Africa and Asia. The sharp hooks of the worms penetrate the small intestinal mucosa and produce bleeding. The worms live for months to years. Infection occurs through the skin, and larval development occurs in the lungs until migration to the trachea and swallowing conducts the worms to the duodenum. Organisms listed in the remaining choices are unlikely to produce significant gastrointestinal hemorrhage.

79
Q

52 A 31-year-old woman had increasingly severe diarrhea 1 week after returning from a trip to Central America. Gross examination of the stools showed mucus and streaks of blood. The diarrheal illness subsided within 4 weeks, but now she has become febrile and has pain in the right upper quadrant of the abdomen. An abdominal ultrasound scan shows a 10-cm, irregular, partly cystic mass in the right hepatic lobe. Which of the following infectious organisms is most likely to produce these findings? A Clostridium difficile B Cryptosporidium parvum C Giardia lamblia D Entamoeba histolytica E Strongyloides stercoralis

A

52 D Diarrhea with mucus and blood in the stools may be caused by several enteroinvasive microorganisms, including Shigella dysenteriae and Entamoeba histolytica. In most cases, the diarrhea is self-limited. The initial episode of diarrhea could have been caused by one of several organisms; however, the occurrence of a liver abscess after an episode of diarrhea most likely results from infection with E. histolytica. Colonic mucosal and submucosal invasion by E. histolytica allows the organisms to access the submucosal veins draining to the portal system and the liver. Clostridium difficile causes pseudomembranous colitis after antibiotic therapy. Dissemination of Cryptosporidium and Strongyloides organisms may occur in immunocompromised patients. Giardia produces a self-limited, watery diarrhea.

80
Q

53 A 27-year-old man has sudden onset of marked abdominal pain. On physical examination, his abdomen is diffusely tender and distended, and bowel sounds are absent. He undergoes surgery, and a 27-cm segment of terminal ileum with a firm, erythematous serosal surface is removed. The microscopic appearance of a section through the excised ileum is shown in the figure. Which of the following additional complications is the patient most likely to develop as a result of this disease process? A Adenocarcinoma B Enterocutaneous fistula C Intussusception D Liver abscess E Mesenteric artery thrombosis

A

53 B The ileum shows chronic inflammation with lymphoid aggregates. The inflammation is transmural, affecting the mucosa, submucosa, and muscularis as shown in the figure. During surgery, inflammation is also observed in the serosa. A deep fissure extending into the muscularis is present. These histologic features are highly suggestive of Crohn disease. Extension of fissures into the overlying skin can produce enterocutaneous fistulas, although enteroenteric fistulas between loops of bowel are more common. Although the risk of adenocarcinoma is increased in Crohn disease, this complication is less common than sequelae of inflammation. Intussusception may occur when there is a congenital or acquired obstruction in the bowel. Hepatic abscess can follow amebic colitis, or other infections. Mesenteric artery thrombosis, typically a complication of atherosclerosis, is unlikely in a 27-year-old man.

81
Q

54 A 30-year-old woman has a 5-year history of recurrent episodes marked by days of abdominal bloating with alternating constipation and diarrhea. She notes hard stools of narrow caliber, low volume mucous diarrhea, and pain in the left lower quadrant. Her symptoms are relieved by defecation, which occurs more frequently now. On physical examination there are no abnormal findings. Laboratory studies including stool for ova and parasites, bacterial pathogens, and fat show no abnormalities. An abdominal CT scan is unremarkable. What is the most likely diagnosis? A Cystic fibrosis B Diverticular disease C Inflammatory bowel disease D Irritable bowel syndrome E Viral gastroenteritis

A

54 D Irritable bowel syndrome (IBS) can be difficult to diagnose because of protean manifestations found in many other conditions. No pathologic or physiologic abnormalities can be identified reliably with IBS. Patients may benefit from behavioral therapies. Placebos may work as well as pharmacotherapies. The lack of an increased stool fat in this case indicates that chronic pancreatitis and cystic fibrosis are unlikely. Diverticular disease is more likely to occur in older adults. Inflammatory bowel disease has both pathologic and radiographic findings. Viral gastroenteritis is unlikely to persist for 5 years.

82
Q

56 A 35-year-old woman has had increasing lower back pain for 5 years. During the past year she also has had arthritic pain involving the knees, hips, and wrists. A stool sample is positive for occult blood. A pelvic radiograph shows changes consistent with sacroiliitis. A colonoscopy is performed, and she undergoes a total colectomy. The figure shows the gross appearance of the colectomy specimen. What is the most likely underlying mechanism of the illustrated condition? A Development of autoantibodies directed against tropomyosin B Development of antimicrobial antibodies that cross react with colonic mucosa C Development of TH17 immune responses D Germline inheritance of the APC gene mutation E Mutations in the NOD2 gene

A

56 C The segment of the colon shows the diffuse and severe ulceration characteristic of ulcerative colitis. The inflammation shown is so severe that areas of mucosal ulceration have produced pseudopolyps or islands of residual mucosa. Ulcerative colitis is a systemic disease; in some patients, it is associated with migratory polyarthritis, ankylosing spondylitis, and primary sclerosing cholangitis. The pathogenesis of ulcerative colitis is unclear, but is most likely mediated by a T-cell response to an unknown antigen (but not a gut infection), leading to an imbalance between T-cell activation and regulation. The TH17 immune response has CD4+ T cells present in the lesions that secrete damaging substances. Autoantibodies against tropomyosin are present, but do not play a pathogenic role in ulcerative colitis. Mutations in the NOD2 gene are linked to Crohn disease, not ulcerative colitis. Inheritance of a germline APC mutation causes familial adenomatous polyposis with a very high risk for colon cancer. Ulcerative colitis also increases the risk for colon cancer, but not secondary to APC gene mutation.

83
Q

57 A 30-year-old woman has suffered intermittent bouts of lower abdominal pain and low-volume diarrhea for the past 2 years. On colonoscopy there is friable mucosa from the rectum to the ascending colon, and a perianal fistula is noted. Biopsies are taken and on microscopic examination show acute and chronic mucosal inflammation with focal erosion. Her stool is negative for ova, parasites, and bacterial pathogens. Which of the following ongoing testing procedures is most useful for long-term follow-up of this woman? A Abdominal CT scanning B Biopsy screening for dysplasia C Genetic mutational analysis for NOD2 D Serologic titers for Saccharomyces E Stool cultures for microbiota

A

57 B The findings in Crohn disease and ulcerative colitis overlap, and in at least 10% of cases it may be impossible to differentiate between them—a so-called indeterminate colitis. Regardless of the exact diagnosis, there is a considerable increase in risk for development of carcinoma 8 to 10 years after disease onset. Surveillance screening can detect dysplasia as a precursor to carcinoma, but would you just remove the colon with the ongoing problem and avoid missing the possible cancer? If you remove the colon, but it turns out to be Crohn disease, it may recur. Extraintestinal manifestations may occur regardless. Doctors like to go for the win with a “cure,” but patients want to avoid potential loss of life or function. The doctor gets to walk away from any outcome, but the patient does not. There are often no easy answers in medicine.

84
Q

58 A 26-year-old man has had intermittent cramping abdominal pain and low-volume diarrhea for 3 weeks. On physical examination, he is afebrile; there is mild lower abdominal tenderness but no masses, and bowel sounds are present. A stool sample is positive for occult blood. The symptoms subside within 1 week. Six months later, the abdominal pain recurs with perianal pain. On physical examination, there is now a perirectal fistula. Colonoscopy shows many areas of mucosal edema and ulceration and some areas that appear normal. Microscopic examination of a biopsy specimen from an ulcerated area shows a patchy acute and chronic inflammatory infiltrate, crypt abscesses, and noncaseating granulomas. Which of the following underlying disease processes best explains these findings? A Amebiasis B Crohn disease C Sarcoidosis D Shigellosis E Ulcerative colitis

A

58 B The clinical and histologic features are consistent with Crohn disease, one of the idiopathic inflammatory bowel diseases. Crohn disease is marked by segmental bowel involvement and transmural inflammation that leads to strictures, adhesions, and fistula. Ulcerative colitis has mucosal involvement extending variable distances from the rectum. In contrast to Crohn disease, the mucosal involvement is diffuse and does not show “skip areas.” Fissures and fistulas are not frequently seen in ulcerative colitis. The findings in Crohn disease and ulcerative colitis overlap, and in at least 10% of cases it may be impossible to differentiate between them—a so-called indeterminate colitis. Generally, crypt abscesses are more typical of ulcerative colitis, and granulomas are more typical of Crohn disease, but these features are not present in most biopsy specimens from patients with either condition. A story is told of an attending physician at an academic medical center who was known to berate students and residents on rounds for not definitively diagnosing ulcerative colitis and Crohn disease. When he retired, incomplete records for patients with idiopathic inflammatory bowel disease were found in his office; the records represented about one sixth of the total cases of inflammatory bowel disease that he had seen. Amebiasis and shigellosis are infectious processes that can cause mucosal ulceration, but they do not produce granulomas or fissures. Sarcoidosis can involve many organs and give rise to noncaseating granulomas; however, involvement of the intestines is uncommon, and sarcoidosis does not give rise to ulcerative disease.

85
Q

59 A clinical study of adult patients with chronic bloody diarrhea is performed. One group of these patients is found to have a statistically increased likelihood for the following: antibodies to Saccharomyces cerevisiae but not anti–neutrophil cytoplasmic autoantibodies, NOD2 gene polymorphisms, TH1 and TH17 immune cell activation, vitamin K deficiency, megaloblastic anemia, and gallstones. Which of the following diseases is this group of patients most likely to have? A Angiodysplasia B Crohn disease C Diverticulitis D Ischemic enteritis E Ulcerative colitis

A

59 B These are findings of idiopathic inflammatory bowel disease most likely to be Crohn disease. The ileal involvement accounts for the vitamin K and vitamin B12 deficiencies as well as disrupted enterohepatic circulation of bile salts predisposing to gallstone formation. The inflammatory response in Crohn disease may result from inappropriate innate immune responses to gut flora, as discussed in the text. Angiodysplasia leads to bleeding from abnormal submucosal vessels, most often in the cecum of older adults. Diverticular disease is common in older persons but results from mechanical, not immune, mechanisms. Severe peripheral atherosclerosis may cause ischemic bowel disease, but this is usually an acute process.

86
Q

60 A 65-year-old woman has a routine health maintenance examination. A stool sample is positive for occult blood. CT scan of the abdomen shows numerous air-filled, 1-cm outpouchings of the sigmoid and descending colon. Which of the following complications is most likely to develop in this patient? A Adenocarcinoma B Bowel obstruction C Pericolic abscess D Malabsorption E Toxic megacolon

A

60 C Colonic diverticulosis may be accompanied by intermittent minimal bleeding and, rarely, by severe bleeding. One or more diverticula may become inflamed (diverticulitis) or, less commonly, may perforate to produce an abscess, peritonitis, or both. Diverticular disease is not a premalignant condition. The diverticula project outward, and even with inflammation, luminal obstruction is unlikely. Malabsorption is not a feature of diverticular disease. Toxic megacolon is an uncommon complication of inflammatory bowel disease.

87
Q

61 The mother of a 4-year-old child notes blood when laundering his underwear. Physical examination reveals a rectal mass. On proctoscopy, there is a smooth-surfaced, pedunculated, 1.5-cm polyp. It is excised and microscopically shows cystically dilated crypts filled with mucin and inflammatory debris, but no dysplasia. What is the most likely diagnosis? A Familial adenomatous polyposis B Gardner syndrome C Juvenile polyp D Lynch syndrome E Peutz-Jeghers syndrome

A

61 C Juvenile polyps are the most common form of hamartomatous polyp. Singly they are likely to be sporadic, and the only complication is rectal prolapse; but when multiple polyps are present, they may be the result of an autosomal dominant syndrome with risk for development of adenocarcinoma. The remaining choices include polyposis syndromes unlikely to appear at this age.

88
Q

62 A 53-year-old woman undergoes a routine checkup. The only abnormal finding is a stool specimen that contains occult blood. Colonoscopy shows a 1.5-cm, solitary, rounded, erythematous polyp on a 0.5-cm stalk at the splenic flexure. The polyp is removed; its histologic appearance is shown in the figure at low (A) and high (B) magnifications. Her colonic lesion is most likely associated with which of the following? A Low risk for development of carcinoma B Inheritance of an abnormal tumor suppressor gene C Presence of similar lesions in the small intestine D History of iron deficiency anemia E Risk for development of endometrial carcinoma

A

62 A The figure shows a solitary pedunculated adenoma of the colon with no evidence of malignancy. High magnification shows a small focus of dysplastic, non–mucinsecreting epithelial cells lining a colonic crypt, giving rise to “tubular” architecture. Such a small (<2 cm), solitary, tubular adenoma is unlikely to harbor a focus of malignancy; a search for metastases is unwarranted. Such colonic adenomas are more likely to occur in older persons; hence the recommendation for colonoscopy screening after age 50. Removing such an adenoma does not leave the chance for further growth of the lesion with possible development of adenocarcinoma. Individuals who inherit a mutant APC gene usually develop hundreds of polyps at a young age; this patient does not need genetic testing for a somatic mutation in the APC gene. Patients with hereditary nonpolyposis colorectal cancer, with multiple polyps present, have an increased risk of endometrial cancer and develop colon cancer at a young age. It is unlikely that the blood loss from a small polyp would be sufficient to cause iron deficiency, although the small amount of blood emanating from colonic polyps and cancers is the rationale to test for fecal occult blood. Peutz-Jehgers syndrome is associated with development of hamartomatous polyps in the small intestine.

89
Q

63 A 70-year-old man has a routine health maintenance examination. On physical examination, there are no remarkable findings, but a stool sample is positive for occult blood. A colonoscopy is performed and shows a 5-cm sessile mass in the upper portion of the descending colon at 50 cm from the anal verge. The histologic appearance at low power of a biopsy specimen of the lesion is shown in the figure. The patient refused further workup and treatment. Five years later, he has constipation, microcytic anemia, and a 5-kg weight loss over 6 months. On surgical exploration, there is a 7-cm mass encircling the descending colon. Which of the following neoplasms is he now most likely to have? A Adenocarcinoma B Non-Hodgkin lymphoma C Carcinoid tumor D Leiomyosarcoma E Mucinous cystadenoma F Villous adenoma

A

63 A The figure shows a large villous adenoma. There is a high probability that large villous adenomas will progress to invasive adenocarcinoma. When they occur in the descending colon, these lesions are annular and cause obstruction. In the colon, non-Hodgkin lymphomas are far less common than adenocarcinomas, and they do not manifest as mucosal sessile masses. Carcinoid tumors are typically small and yellowish, and most grow slowly. Leiomyosarcomas are rare; they produce large bulky masses, but they do not arise on the mucosa. Mucinous cystadenomas are cystic and are more likely to arise in an ovary or in the pancreas. The original lesion in this patient was a villous adenoma.

90
Q

64 A 19-year-old man is advised to see his physician because genetic screening has detected a disease in other family members. On physical examination, a stool sample is positive for occult blood. A colonoscopy is performed, followed by a colectomy. The figure shows the gross appearance of the mucosal surface of the colectomy specimen. Microscopic examination shows these lesions are tubular adenomas. Molecular analysis of this patient’s normal fibroblasts is most likely to show a mutation in which of the following genes? A APC B MLH1 C KRAS D NOD2 E p53

A

64 A This young patient’s colon shows hundreds of polyps. This is most likely a case of familial adenomatous polyposis (FAP) syndrome, which results from inheritance of one mutant copy of the APC tumor-suppressor gene (a few FAP cases are associated with DNA mismatch repair genes). Every somatic cell of this patient most likely has one defective copy of the APC gene. Polyps are formed when the second copy of the APC gene is lost in many colon epithelial cells. Without treatment, colon cancers arise in 100% of these patients because of accumulation of additional mutations in one or more polyps, typically before 30 years of age. Patients with a gene for hereditary nonpolyposis colorectal carcinoma, such as MLH1 and MSH2, also have an inherited susceptibility to develop colon cancer, but in contrast to patients with FAP, they do not develop numerous polyps. Sporadic colon cancers may have CpG island hypermethylation along with KRAS mutations, whereas others have p53 mutations, but the somatic cells of patients with these cancers do not show abnormalities of these genes. NOD2 mutations are linked with Crohn disease.

91
Q

65 A 44-year-old woman has had increasing abdominal distention for the past 6 weeks. On physical examination, there is an abdominal fluid wave, and bowel sounds are present. Paracentesis yields 1000 mL of slightly cloudy serous fluid. Cytologic examination of the fluid shows malignant cells consistent with adenocarcinoma. Molecular analysis of these cells shows an MSH2 gene mutation with microsatellite instability. Her medical history indicates that she has had no major medical illnesses and no surgical procedures. Her sister was diagnosed with endometrial cancer and her brother had carcinoma of the stomach. Which of the following conditions is the most likely cause of this patient’s symptoms? A Angiodysplasia B Crohn disease C Diverticulosis D Lynch syndrome E Peptic ulcer disease

A

65 D Of the conditions listed, the one most likely to lead to adenocarcinoma in a patient of this age is hereditary nonpolyposis colorectal cancer, or Lynch syndrome. Crohn disease is unlikely because the patient has not had prior serious illness, and Crohn disease of long duration is unlikely to remain asymptomatic. Although adenocarcinoma may complicate Crohn disease, it does not occur as frequently as in ulcerative colitis. This explains why colectomy is often performed for ulcerative colitis, but bowel resections are avoided, if possible, in Crohn disease. The other conditions listed are not premalignant.

92
Q

66 A 33-year-old man has a routine health maintenance examination. A stool sample is positive for occult blood. On colonoscopy, a 6-cm ulcerative lesion is seen projecting into the cecum. There are three smaller sessile lesions from 1 to 3 cm in size. The microscopic appearance of a section of the ulcerated lesion is shown in the figure. The smaller lesions are reported as sessile serrated adenomas. Which of the following molecular biological events is thought to be most critical in the development of such lesions? A Amplification of ERBB2 gene B Defective DNA mismatch repair gene C Germline transmission of a defective RB1 gene D Overexpression of E-cadherin gene E Translocation of retinoic acid receptor alpha gene

A

66 B The lesion is an adenocarcinoma, showing irregular glands infiltrating the muscle layer. Such a lesion in a 30-yearold man strongly indicates a hereditary predisposition. One hereditary form of cancer is called hereditary nonpolyposis colorectal cancer (HNPCC) and results from defective DNA mismatch repair genes. As a result, mutations accumulate in microsatellite repeats (microsatellite instability) that lead to loss of transforming growth factor beta (TGF-β) receptormediated control of colonic epithelial cell proliferation and loss of proapoptotic BAX protein enhancing survival of these transformed cells. He could have taken NSAIDs that inhibit COX-2 expressed in most colonic adenomas and carcinomas. In contrast to familial adenomatous polyposis syndrome, HNPCC does not lead to the development of hundreds of polyps in the colon. Detection of ERBB2 (HER2/NEU) expression is important in breast cancers. Germline inheritance of the tumor suppressor gene RB1 predisposes to retinoblastoma and osteosarcoma, not colon carcinoma. E-cadherin is required for intercellular adhesion; its levels are reduced, not increased, in carcinoma cells. Translocation of the retinoic acid receptor alpha gene is characteristic of acute promyelocytic leukemia.

93
Q

67 A 73-year-old man has noted a change in bowel habits for the past year. Defecation is more difficult and the caliber of stools has decreased. On physical exam, there are no abnormal findings except for stool positive for occult blood. Colonoscopy is performed for the first time in this man, followed by colonic resection with the gross appearance shown in the figure. Which of the following molecular abnormalities has most likely led to these findings? A Acquired APC gene mutation B Homozygous loss of PTEN gene C Inactivation of the RB1 protein by HPV-16 D Mutation in a DNA mismatch repair gene E Tyrosine kinase activation with KIT mutation

A

67 D The figure shows an encircling mass that is typical of adenocarcinoma of the descending colon. Such cancers likely to obstruct, but they can also bleed a small amount over months to years, causing iron deficiency anemia. The APC gene, a negative regulator of β-catenin in the WNT signaling pathway, is associated with familial adenomatous polyposis syndrome and most sporadic colon cancers, as in this case. This pathway also is known as the adenoma-carcinoma sequence because the carcinomas develop through an identifiable series of molecular and morphologic steps. Loss of the PTEN tumor suppressor gene is seen in endometrial carcinomas not associated with colon carcinoma and with some hamartomatous polyps of the colon. Evidence for an additional cancer, such as an endometrial cancer, would suggest an inherited mutation in one of the DNA mismatch repair genes, such as MSH2 and MLH1. Homozygous loss of these genes can give rise to right-sided colon cancer and endometrial cancer. Such a mutation is typically associated with microsatellite instability. Infection with some strains of human papillomavirus leads to RB1 protein inactivation and development of cervical carcinoma. Mutation with activation of KIT tyrosine kinase activity occurs in gastrointestinal stromal tumors, which respond well to treatment with imatinib mesylate, a tyrosine kinase inhibitor also used to treat chronic myelogenous leukemia.

94
Q

68 A 20-year-old woman in her ninth month of pregnancy has increasing pain on defecation and notices bright red blood on the toilet paper. She has had no previous gastrointestinal problems. After she gives birth, the rectal pain subsides, and there is no more bleeding. Which of the following is the most likely cause of these findings? A Angiodysplasia B Hemorrhoids C Intussusception D Ischemic colitis E Volvulus

A

68 B Hemorrhoids are a common problem that can stem from any condition that increases venous pressure and causes dilation of internal or external hemorrhoidal veins above and below the anorectal junction. Angiodysplasia of the colon leads to intermittent hemorrhage, typically in older individuals. Ischemic colitis is rare in young individuals because the most common underlying cause (severe atherosclerotic disease involving mesenteric vessels) occurs in older patients. Intussusception and volvulus are rare causesof mechanical bowel obstruction; they occur suddenly in adults and are surgical emergencies.

95
Q

69 A 20-year-old woman has had nausea and vague lower abdominal pain for the past 24 hours, but now the pain has become more severe. On physical examination, the pain is worse in the right lower quadrant, and there is rebound tenderness. A stool sample is negative for occult blood. Abdominal plain film radiographs show no free air. The result of a serum pregnancy test is negative. Which of the following laboratory findings is most useful to aid in the diagnosis of this patient? A Entamoeba histolytica cysts in the stool B Hyperamylasemia C Hypernatremia D Increased serum alkaline phosphatase E Increased serum carcinoembryonic antigen F Neutrophilia with left shift

A

69 F Acute appendicitis can be accompanied by an elevated WBC count with neutrophilia and left shift. This is helpful but not decisive, and the decision to operate must be based on clinical judgment. Amebiasis is most likely associated with a history of diarrhea, often with blood in the stool. Hyperamylasemia occurs in acute pancreatitis. Diarrhea with fluid loss and dehydration can lead to hypernatremia. The serum carcinoembryonic antigen level may be increased in patients with colonic cancers; however, this test is not specific for colon cancer. The alkaline phosphatase level may be increased in biliary tract obstruction.

96
Q

70 A 53-year-old woman has increasing abdominal girth for the past 2 years. On physical examination she has abdominal distension. An abdominal CT scan shows multiple nodules on peritoneal surfaces along with low attenuation mucinous ascites. Paracentesis is performed and cytologic examination of the fluid obtained shows well-differentiated columnar cells with minimal nuclear atypia. Where did this proliferative process most likely arise in this woman? A Appendix B Jejunum C Ileum D Pancreas E Stomach

A

70 A Pseudomyxoma peritonei (PP) is described here. It may arise from low-grade mucinous adenocarcinoma of the appendix, which may be so differentiated that it resembles an appendiceal mucocele. However, PP tends to recur. In women, PP needs to be distinguished from mucinous tumors of the ovary. Mucinous tumors may also arise in the pancreas, but are less likely to disseminate through the peritoneal cavity. Malignancies arising in the small intestine are rare. Mucinproducing malignancies of the stomach are most likely to have a signet ring cell pattern and diffusely infiltrate the gastric wall.

97
Q

71 A 59-year-old man with a lengthy history of chronic alcoholism has noticed increasing abdominal girth for the past 6 months. He has had increasing abdominal pain for the past 2 days. On physical examination, his temperature is 38.2° C. Examination of the abdomen shows a fluid wave and prominent caput medusae over the skin of the abdomen. There is diffuse abdominal tenderness. An abdominal plain film radiograph shows no free air. Paracentesis yields 500 mL of cloudy yellow fluid. Gram stain of the fluid shows gram-negative rods. Which of the following is the most likely diagnosis? A Appendicitis B Collagenous colitis C Diverticulitis D Ischemic colitis E Pseudomembranous colitis F Spontaneous bacterial peritonitis

A

71 F Spontaneous bacterial peritonitis is an uncommon complication found in about 10% of adult patients with cirrhosis of the liver and ascites. The ascitic fluid provides an excellent culture medium for bacteria, which can invade the bowel wall or spread hematogenously to the serosa. Spontaneous bacterial peritonitis also can occur in children, particularly children with nephrotic syndrome and ascites. The most common organism cultured is Escherichia coli. Appendicitis has a peak incidence in younger patients; the pain is often (but not always) more localized in the right lower quadrant, and ascites is usually absent. Appendicitis is not related to alcoholism. Collagenous colitis is uncommon; it most often leads to watery diarrhea in middle-aged women. Diverticulitis with rupture could produce peritonitis, but there is typically no ascites, and diverticulitis is not related to alcoholism. Ischemic colitis may produce infarction with rupture and peritonitis, but ascites is usually lacking, and individuals with chronic alcoholism are unlikely to have marked atherosclerosis. Pseudomembranous colitis is a complication of antibiotic therapy.

98
Q

18 A 38-year-old woman has had nausea for 6 months. She reports no vomiting or diarrhea. On physical examination, there are no remarkable findings. Upper gastrointestinal endoscopy shows diffuse gastric mucosal erythema with focal mucosal erosions, but no ulcerations. The esophageal and duodenal mucosal surfaces appear normal. Gastric biopsies are obtained and microscopic examination shows focal mucosal hemorrhage, loss of the surface epithelium, and increased numbers of neutrophils, lymphocytes, and plasma cells in an edematous mucosa. No Helicobacter pylori organisms are seen. Laboratory studies show a normal serum gastrin level. Which of the following pharmacologic agents is most likely to produce these findings? A Aspirin B Chlorpromazine C Cimetidine D Clindamycin E Omeprazole

A

18 A These findings are consistent with an acute gastritis. If significant inflammation is not present, then the term gas tropathy is used. Heavy consumption of ethanol is probably the most common cause, but aspirin and nonsteroidal antiinflammatory drugs (NSAIDs), smoking, and chemotherapy agents can produce the same findings. NSAIDs can be cofactors in peptic ulcer disease. Chlorpromazine (used to treat nausea) does not have the same association. Cimetidine and omeprazole are used to treat peptic ulcer disease by reducing gastric acid production, increasing the serum gastrin. Cimetidine is an H2 receptor blocker, and omeprazole is a proton pump inhibitor. Clindamycin is a broad-spectrum antibiotic that may alter flora in the lower gastrointestinal tract.

99
Q

32 A 61-year-old man with increasing fatigue, early satiety, and nausea for 5 months vomited dark granular material yesterday. Endoscopy reveals a large ulcerated mass in the gastric fundus. Biopsies are taken and microscopically the mass is composed of spindle cells that are positive for c-Kit with immunohistochemical staining. Mitoses are frequent. Gastrectomy is performed, and the 10-cm circumscribed mass arises from the gastric wall. Which of the following therapies is most likely to be a useful adjunct in treatment of his disease? A Amoxicillin B Azathioprine C Cyclophosphamide D Imatinib E Prednisone F Radiation

A

32 D This gastrointestinal stromal tumor (GIST) is derived from the interstitial cell of Cajal, and hence of mesenchymal origin. Those arising in the stomach may be less aggressive than those arising in the intestine, but most are KIT positive and amenable to tyrosine kinase inhibitor therapy. Some GISTs may have mutations in platelet-derived growth factor receptor A (PDGFRA). Antibiotic therapy to obliterate Helicobacter pylori infection may be useful in treating MALTomas. Azathioprine and corticosteroids may be employed in treating inflammatory bowel disease, but the greatest risk for malignancy with inflammatory bowel disease is adenocarcinoma, particularly of the colon. Cyclophosphamide is a chemotherapy agent not employed in treating GISTs. Radiotherapy is not generally effective against mesenchymal malignancies.

100
Q

55 A 49-year-old woman has had abdominal cramps and diarrhea with six stools per day for the past month. She has a history of similar episodes of self-limited pain and diarrhea, which have occurred multiple times during the past 20 years. Each episode lasts about 2 weeks and resolves without treatment. Findings on physical examination are unremarkable, but a stool sample is positive for occult blood. Laboratory studies show no ova or parasites in the stool. Colonoscopy shows diffuse and uninterrupted mucosal inflammation and superficial ulceration extending from the rectum to the ascending colon. Colonic biopsy specimens from the area show the findings in the figure. She is at greatest risk for developing which of the following complications? A Adenocarcinoma B Diverticulitis C Fat malabsorption D Perirectal fistula formation E Primary biliary cirrhosis F Pseudomembranous colitis

A

55 A The figure shows a diffuse, predominantly mononuclear infiltrate in the lamina propria along with a crypt abscess. Ulcerative colitis can lead to relapsing and remitting episodes of low volume diarrhea containing blood and mucus and diffuse inflammation and ulceration of the rectal and colonic mucosa. One of the most dreaded complications of ulcerative colitis is the development of colonic adenocarcinoma. There is a twentyfold to thirtyfold higher risk in patients who have had ulcerative colitis for 10 or more years compared with control populations. Diverticulitis can produce abdominal pain and blood in the stool, but there is no association with ulcerative colitis. Fat malabsorption usually does not occur in ulcerative colitis because the ileum often is not involved. Perirectal fistula formation is more typical of Crohn disease, in which there is transmural inflammation. Ulcerative colitis is associated with several extraintestinal manifestations, including sclerosing cholangitis, but it has no relationship to primary biliary cirrhosis. Pseudomembranous colitis is caused by Clostridium difficile infections associated with broad-spectrum antibiotic treatment.